*NURSING > TEST BANK > NUR 111 APEA_Questions__1_HELPFUL (Diabetes, 87-year-old)/ (196 pages of Q&A) QUIZ BANK | Already GR (All)

NUR 111 APEA_Questions__1_HELPFUL (Diabetes, 87-year-old)/ (196 pages of Q&A) QUIZ BANK | Already GRADED A.

Document Content and Description Below

Diabetes, 87-year-old Cardiovascular Disorders Benazepril should be discontinued immediately if: pregnancy occurs. A patient is diagnosed with mild heart failure (HF). What drug listed below wo... uld be a good choice for reducing morbidity and mortality long term? Metoprolol A patient taking an ACE inhibitor should avoid: potassium supplements. An 87-year-old has history of symptomatic heart failure. He presents today with lower extremity edema and mild shortness of breath with exertion. In addition to a diuretic for volume overload, what other medication should he receive today? ACE inhibitor A 42-year-old hypertensive patient was given a thiazide diuretic 4 weeks ago for treatment of primary hypertension. On his return visit today, he reports feeling weak and tired. What should the NP consider to evaluate the weakness and fatigue? Potassium level Which class of medication is frequently used to improve long- term outcomes in patients with systolic dysfunction? ACE inhibitors Orthostatic hypotension can be diagnosed in an older adult if the systolic blood pressure decreases: more than 20 points within 3 minutes after rising. A 74-year-old patient has peripheral artery disease (PAD). Which item listed below is an important nonmodifiable risk factor for Diabetes PAD? The lipid particle with the greatest atherogenic effect is: LDL. Which medication could potentially exacerbate heart failure (HF)? Naproxen Where would the murmur associated with mitral regurgitation best be auscultated? Mitral listening point A 43-year-old Hispanic male has an audible diastolic murmur best heard in the mitral listening point. There is no audible click. His status has been monitored for the past 2 years. This murmur is probably: mitral stenosis. An independent 82-year-old male patient is very active but retired last year. His total cholesterol and LDLs are moderately elevated. How should the NP approach his lipid elevation? Treatment is based on expected length of life A patient who has diabetes presents with pain in his lower legs when he walks and pain resolution with rest. When specifically asked about the pain in his lower leg, he likely will report pain: in the calf muscle. A 75-year-old patient with longstanding hypertension takes an ACE inhibitor and a thiazide diuretic daily. He has developed dyspnea on exertion and peripheral edema over the past several days. This probably indicates: development of heart failure (HF). An ACE inhibitor is specifically indicated in patients who have: hypertension, diabetes with proteinuria, heart failure. The correlation between blood pressure and age greater than 60 years is that as age increases: diastolic blood pressure decreases. A 77-year-old patient has had an increase in blood pressure since the last exam. The blood pressure readings are provided. If medication is to be started on this patient, what would be a good first choice? Calcium channel blocker Mrs. Brandy is having contrast dye next week for a heart catheterization. What drug does NOT need to be stopped prior to her catheterization? Losartan Older adults have a unique blood pressure pattern. Which blood pressure reading below reflects this pattern? 160/60 mmHg A 91-year-old female with longstanding history of chronic heart failure has renal and liver studies that have slowly worsened over the past year. This probably indicates: target organ damage secondary to heart failure. The usual clinical course of mitral valve prolapse: is benign. Which antibiotic should be used with caution if an older patient has cardiac conduction issues? Ciprofloxacin A patient with aortic stenosis has been asymptomatic for decades. On routine exam, he states that he has had some dizziness associated with activity but no chest pain or shortness of breath. The best course of action for the nurse practitioner is to: refer to cardiology. The nurse practitioner is caring for an independent 74-year-old female who had acute coronary syndrome (ACS) about 6 weeks ago. What medications should be part of her regimen unless there is a contraindication? ACE, ASA, beta blocker, and statin Which hypertensive patient is most likely to have adverse blood pressure effects from excessive sodium consumption? 70-year-old African American male A common side effect of thiazide diuretics is: erectile dysfunction. The carotid arteries are auscultated for bruits because: this is indicative of generalized atherosclerosis. Which choice below would be the best choice for an 80-year-old patient whose blood pressure is 172/72 mm Hg? Amlodipine Which item below represents the best choice of antihypertensive agents for the indicated patient? Lisinopril in a 35-year-old Caucasian male A patient with hypertension has been diagnosed with gout. Which home medication may have contributed to this episode of gout? Furosemide You are managing the warfarin dose for an older adult with a prosthetic heart valve. Which situation listed requires that warfarin be discontinued now? INR of 8, no significant bleeding Which of the following medications may have an unfavorable effect on a hypertensive patient's blood pressure? Naproxen Which study would be most helpful in evaluating the degree of hypertrophy of the atrium or ventricle? Echocardiography A patient has had poorly controlled hypertension for more than 10 years. Indicate the most likely position of his point of maximal impulse (PMI): 5th ICS, left of MCL. Drugs that target the renin-angiotensin-aldosterone system are particularly beneficial in patients who have: diabetic nephropathy. A patient taking atorvastatin for newly diagnosed dyslipidemia complains of fatigue, weakness, and muscle aches in his lower back, arms, and legs for the past three days. It has not improved with rest. How should this be evaluated? Stop the atorvastatin immediately. How often should lipids be screened in patients who are 65 years and older if they have lipid disorders or cardiovascular risk factors? Annually A patient who takes HCTZ 25 mg daily has complaints of muscle cramps. He probably has: hypokalemia. An older adult has renal insufficiency, hypertension, osteoarthritis, hypothyroidism, and varicose veins. Which medication should be avoided? NSAIDs An immune response to Group A Streptococcal infections involving the heart is: rheumatic fever. Warfarin treatment is greatly influenced by a patient's food and medication intake. Which group listed can potentially decrease INR (International Normalized Ratio) in an outpatient who takes warfarin? Sucralfate and cholestyramine A patient with newly diagnosed heart failure has started fosinopril in the last few days. She has developed a cough. What clinical finding can help distinguish the etiology of the cough as heart failure and not related to fosinopril? It is wet and worse with recumbence. You have been asked to evaluate a heart murmur in a pregnant patient. Can a 3D echocardiogram be safely used to evaluate her? Yes, this is perfectly safe. A patient with hypertension describes a previous allergic reaction to a sulfa antibiotic as "sloughing of skin" and hospitalization. Which medication is contraindicated in this patient? Hydrochlorothiazide In older adults, the three most common ailments are: hearing loss, hypertension, arthritis. The major difference between varicose veins and arteriosclerosis is the: vessels affected. A patient with mitral regurgitation (MR) has developed the most common arrhythmia associated with MR. The intervention most likely to prevent complications from this arrhythmia is: anticoagulation. A patient taking an angiotensin receptor blocker (ARB) should avoid: potassium supplements. Mr. Brown a 45-year-old African American male has the following lab values. What should the nurse practitioner do next? A thyroid stimulating hormone (TSH) level Which choice below characterizes a patient who has aortic regurgitation? Long asymptomatic period followed by exercise intolerance, then dyspnea at rest Pharmacologic treatment for older adults with hypertension should be initiated: for any type of hypertension. The most common indicator of end-organ damage in adolescents with hypertension is: left ventricular hypertrophy. A decrease in blood pressure can occur in men who take sildenafil (Viagra) and: any antihypertensive medication. A 75-year-old patient with longstanding hypertension takes a combination ACE inhibitor/thiazide diuretic and amlodipine daily. Today his diastolic blood pressure and heart rate are elevated. He has developed dyspnea on exertion and peripheral edema over the past several days. These symptoms likely demonstrate: development of heart failure. Tables are used for determination of maximum blood pressure values for adolescents. How are these blood pressure values established for adolescents? Height percentile, gender, and age Which group of medications would be detrimental if used to treat a patient who has heart failure (HF)? Fosinopril, HCTZ, verapamil Mr. Holbrook, a 75-year-old male, is a former smoker with a 30 pack-year history. He has come in today for an annual exam. He walks daily for 25 minutes, has had intentional weight loss, and has a near normal BMI. On examination, the patient is noted to have an absence of hair growth on his lower legs. Which statement is true regarding this patient? This might indicate disease in the lower extremities. Which test listed below may be used to exclude a secondary cause of hyperlipidemia in a patient who has elevated lipids? TSH An overweight 76-year-old female with a recent onset of diabetes has longstanding hypertension and hyperlipidemia. She has developed atrial fibrillation. The nurse practitioner knows that this patient is at risk for: heart failure (HF). A medication that may produce exercise intolerance in a patient who has hypertension is: metoprolol. A patient has shortness of breath. If heart failure is the etiology, which test demonstrates the highest sensitivity in diagnosing this? B type natriuretic peptide (BNP) According to the National Heart, Lung and Blood Institute, which characteristic listed below is a coronary heart disease (CHD) risk equivalent; that is, which risk factor places the patient at a CHD risk similar to a history of CHD? Diabetes mellitus A 75-year-old patient who has aortic stenosis wants to know what symptoms indicate worsening of his stenosis. The nurse practitioner replies: shortness of breath and syncope. A 55-year-old male is obese, does not exercise, and has hyperlipidemia. His average blood pressure is 150/90 mmHg. How should he be managed today? Lifestyle modifications are appropriate. A patient will be screened for hyperlipidemia via a serum specimen. He should be told: to fast for 12-14 hours. Which patient is most likely to have mitral valve prolapse? A 30-year-old female with no cardiac history A 40-year-old African American patient has blood pressure readings of 175/100 mmHg and 170/102 mmHg. What is a Initiate 5 mg amlodipine daily (usual dose is 5-10 mg reasonable plan of care for this patient today? daily). Besides hypertension, which risk factor most contributes to development of an abdominal aortic aneurysm? Cigarette smoking A 50-year-old patient with hypertension has taken hydrochlorothiazide 25 mg daily for the past 4 weeks. How should the nurse practitioner proceed? Add a drug from another class to the daily 25 mg hydrochlorothiazide. Three of the following medications warrant monitoring of potassium levels. Which one does NOT? Amlodipine The most common arrhythmia resulting from valvular heart disease is: atrial fibrillation. Classic symptoms of deep vein thrombosis (DVT) include: swelling, pain, and discoloration in lower extremity. A 65-year-old male patient has the following lipid levels. What class of medications is preferred to normalize his lipid levels and reduce his risk of a cardiac event? HMG CoA reductase inhibitors A 28-year-old has a Grade 3 murmur. Which characteristic indicates a need for referral? A fixed split Which mitral disorder results from redundancy of the mitral valve's leaflets? Mitral valve prolapse Mr. Smith is a 72-year-old patient who takes warfarin for chronic atrial fibrillation. His INR and CBC results are provided. The nurse practitioner should: stop the warfarin today and repeat the INR tomorrow. In order to reduce lipid levels, statins are most beneficial when taken: in conjunction with diet and exercise. A child's resting heart rate is expected to be between 60 and 100 beats per minute once he reaches: 10 years of age. An older adult who has hypertension and angina takes multiple medications. Which one of the following decreases the likelihood of his having angina? Beta blocker At what age should initial blood pressure screening take place? 3 years Which patient could be expected to have the highest systolic blood pressure? A 75-year-old male An older adult who has hypertension also has osteoporosis. Which antihypertensive agent would have the secondary effect of improving her osteoporosis? A thiazide diuretic Mr. Daigle is an 80-year-old patient who takes warfarin for chronic atrial fibrillation. The nurse practitioner should: stop warfarin for the next two days and repeat the INR on day 3. A common, early finding in patients who have chronic aortic regurgitation (AR) is: an hypertrophied left ventricle. A nurse practitioner has not increased the dosage of an antihypertensive medication even though the patient's blood pressure has remained >140/90 mmHg. This might be described as: clinical inertia. How often should blood pressure be measured in a child who is 3 years old? It should be measured annually. Risk assessment for dyslipidemia should begin at: 2 years. Ramipril has been initiated at a low dose in a patient with heart failure. What is most important to monitor in about 1 week? Potassium level An 80-year-old patient with longstanding hypertension takes Monopril and HCTZ for hypertension. His most recent blood pressures are listed. What should be done about his blood pressure? Add a calcium channel blocker A patient taking candesartan for treatment of hypertension should avoid: potassium supplements. Pharmacologic treatment for children who have hypertension should be initiated for: diabetics with hypertension. Many factors can contribute to the risk of congenital heart disease. Which maternal disease carries a higher risk of transposition of the great vessels (TGA), ventricular septal defect (VSD), and hypertrophic cardiomyopathy? Diabetes Which test below is most cost-effective to screen for abdominal aortic aneurysm? Abdominal ultrasound A patient with poorly controlled hypertension and history of myocardial infarction 6 years ago presents today with mild shortness of breath. He takes quinapril, ASA, metoprolol, and a statin daily. What symptom is NOT indicative of heart failure? Headache A patient who has mitral valve prolapse (MVP) reports chest pain and frequent arrhythmias. In the absence of other underlying cardiac anomalies, the drug of choice to treat her symptoms is: metoprolol. A 25-year-old patient has aortic stenosis (AS). The etiology of his AS is probably: congenital. A young child has an audible murmur. The nurse practitioner describes it as a grade 4 murmur. How should this be managed? The child should be referred to cardiology. A characteristic of an ACE inhibitor-induced cough is that it: usually begins within 2 weeks of starting therapy. The valve most commonly involved in chronic rheumatic heart disease is the: mitral. A 75-year-old has isolated systolic hypertension. She started on amlodipine 4 weeks ago. She states that since then, she has developed urinary incontinence. What is the nurse practitioner's assessment? It is probably related to amlodipine. It is probably related to amlodipine. Most hypertension in preadolescents and children is: secondary hypertension. The diagnosis of mitral valve prolapse can be confirmed by: echocardiography. Mrs. Jones is an 85-year-old who has average blood pressures of 170/70 mmHg. Which agent would be a good starting medication to normalize her blood pressure? Amlodipine Which laboratory abnormality may be observed in a patient who takes lisinopril? Increased potassium level Which medication listed below could potentially exacerbate heart failure in a susceptible patient? Metoprolol A congenital heart abnormality often discovered during the newborn period is coarctation of the aorta. How is this assessed? By comparing upper and lower extremity blood pressures A 64-year-old male has been your patient for several years. He is a former smoker. He presents to your clinic with complaints of fatigue and "just not feeling well" for the last few days. His exam is normal. His blood pressure is well controlled. His medication list, most recent lipid panel, and today's vital signs are provided. What should be done next? Order a CBC, metabolic panel, TSH, and urine analysis. An 80-year-old female who is otherwise well has the following blood pressure readings. How should she be managed pharmacologically? Calcium channel blocker Which infant feeding behavior is least likely related to congenital heart disease (CHD)? Infants that burp frequently when feeding Dermatological Disorders A 70-year-old is diagnosed with multiple cherry angiomas. The nurse practitioner knows that: these may bleed profusely if ruptured. A patient is found to have koilonychia. What laboratory test would be prudent to perform? Complete blood count Most cases of atopic dermatitis exacerbation are treated with: topical steroids. Which vehicle is least appropriate in a patient who has atopic dermatitis? Lotions A 23-year-old male appears in clinic with the following lesion on his trunk. This lesion is usually associated with: pityriasis rosea. The nurse practitioner examines a patient who has had poison ivy for 3 days. She asks if she can spread it to her family members. The nurse practitioner replies: “No, transmission does not occur from the blister’s contents.” A patient will be taking oral terbinafine for fingernail fungus. The NP knows that: terbinafine is an inhibitor of the CYP 2D6 enzymes. Mr. Johnson is a 74-year-old who presents with a pearly-domed, nodular-looking lesion on the back of the neck. It does not hurt or itch. What is a likely etiology? Basal cell carcinoma A patient reports that he found a tick on himself about 2 weeks ago. He presents today with a red circle and a white center near where he remembers the tick bite. He did not seek treatment at that time. Today he complains of myalgias and arthralgias. Which laboratory test can be used to help diagnose Lyme disease? ELISA A patient with a positive history of a tick bite about 2 weeks ago and erythema migrans has a positive ELISA for Borrelia burgdorferi. The Western blot is positive. How should he be managed? He should receive doxycycline for Lyme disease. A 74-year-old male patient has sustained a laceration to his foot. His last tetanus shot was more than 10 years ago. He has completed the primary series. What should be recommended? Tetanus, diphtheria, and acellular pertussis (Tdap) A 71-year-old female presents with a vesicular rash that burns and itches. Shingles is diagnosed. An oral antiviral: should be started within 72 hours of the onset of symptoms. A microscopic examination of the sample taken from a skin lesion indicates hyphae. What type of infection might this indicate? Fungal Topical 5-fluorouracil (5-FU) is used to treat: basal cell carcinoma. A patient was burned with hot water. He has several 2-3 cm fluid-filled lesions. What are these termed? Bullae Which test is NOT suitable to diagnose shingles if the clinical presentation is questionable? Complete blood count (CBC) A patient has been in the sun for the past few weeks and has developed darkened skin and numerous 3-6 mm light-colored, flat lesions on his trunk. What is the likely etiology? Tinea versicolor Which of the following antibiotics may increase the likelihood of photosensitivity? Ciprofloxacin A low-potency topical hydrocortisone cream would be most appropriate in a patient who has been diagnosed with: atopic dermatitis. A 16-year-old male has nodulocystic acne. What might have the greatest positive impact in managing his acne? Isotretinoin (Accutane) The agent commonly used to treat patients with scabies is permethrin. How often should it be applied to eradicate scabies? Once A 40-year-old female patient presents to the clinic with multiple, painful reddened nodules on the ulcerative colitis. anterior surface of both legs. She is concerned. These are probably associated with her history of: A patient presents with plaques on the extensor surface of the elbows, knees, and back. The plaques are erythematous and thick, silvery scales are present. This is likely: plaque psoriasis. Which of the following skin lesions in an older adult is a premalignant condition? Actinic keratosis A patient has seborrheic dermatitis. Which vehicle would be most appropriate to use in the hairline area to treat this? Foam A 16-year-old has been diagnosed with Lyme disease. Which drug should be used to treat him? Doxycycline A patient exhibits petechiae on both lower legs but has no other complaints. How should the NP proceed? Order a CBC Which chronic skin disorder primarily affects hairy areas of the body? Seborrheic dermatitis A skin lesion that is a solid mass is described as a: papule. An older adult patient has been diagnosed with shingles on the right lateral aspect of her trunk. It initially appeared yesterday. It is very painful. How should she be managed? An oral antiviral agent and pain medication A patient has been diagnosed with scabies. What is the medication of choice to treat this? Permethrin Patients with atopic dermatitis are likely to exhibit: itching. The primary therapeutic intervention for patients who present with hives is: antihistamines. Which of the following lesions never blanches when pressure is applied? Purpura or petechiae The most common form of skin cancer is: basal cell carcinoma. Which of the following areas of the body has the greatest percutaneous absorption? Genitalia A patient presents to the minor care area of the emergency department after being bitten by a dog. The patient states that the dog had a tag around his neck and had been seen roaming around the neighborhood for days before the patient was bitten. The dog did not exhibit any odd behavior. How should this be managed? Report the bite to animal control and administer appropriate medical care. A 28-year-old has thick, demarcated plaques on her elbows. Which features are suggestive of psoriasis? Silvery scales that are not pruritic A patient is diagnosed with tinea pedis. A microscopic examination of the sample taken from the infected area would likely demonstrate: hyphae. The most common place for basal cell carcinoma to be found is the: face. A 60-year-old patient is noted to have rounding of the distal phalanx of the fingers. What might have caused this? Hepatic cirrhosis A 68-year-old female adult with pendulous breasts complains of “burning” under her right breast. The nurse practitioner observes a malodorous discharge with mild maceration under both breasts. What is this? Intertrigo A skin disorder has a hallmark finding of silvery scales. What word below describes this common condition? Chronic A wound has the following characteristics; partial thickness loss of dermis, a shallow open ulcer with red/pink bed, and no evidence of sloughing. What stage of pressure ulcer does this describe? Stage II A patient has been diagnosed with MRSA. She is allergic to sulfa. Which medication could be used to treat her? Doxycycline A patient has a “herald patch” and is diagnosed with pityriasis rosea. Where is the “herald patch” most commonly found? On the chest The American Cancer Society uses an ABCDE mnemonic to help patients develop awareness of suspicious skin lesions. What does the “B” represent? Border A patient calls your office. He states that he just came in from the woods and discovered a tick on his upper arm. He states that he has removed the tick and the area is slightly red. What should he be advised? No treatment is needed. An example of a first-generation cephalosporin used to treat a skin infection is: cephalexin. A skin lesion fluoresces under a Wood’s lamp. What microscopic finding is consistent with this? Hyphae A patient with a primary case of scabies was probably infected: 3-4 weeks ago. A patient has 10 cm of well demarcated erythema on his lower leg that is raised and warm to touch. He had an abrupt onset of lower leg pain, and fever that began 36 hours ago. What is this? Erysipelas An adolescent takes isotretinoin for nodulocystic acne. She is on oral contraceptives. Both were prescribed by the dermatologist. The adolescent presents to your clinic with a sinus infection. Her temperature is 99.5° F and her blood pressure is 160/100 mmHg. How should this be managed? Call the dermatologist to report the elevated BP A 74-year-old woman is diagnosed with shingles. The NP is deciding how to best manage her care. What should be prescribed? An oral antiviral agent Hand-foot-and-mouth disease and herpangina: are viral infections caused by Coxsackie viruses. A 23-year-old male appears in clinic with the following nonpruritic lesion on his trunk. He first noticed this about 3 days ago. The lesion is probably: a herald patch. A patient has suspected scarlet fever. He likely has a sandpaper rash and: a positive rapid Strept test. A patient who is at high risk for skin cancer should: examine his skin monthly for changes. A 9-year-old female has presented to your clinic because of a rash on the left, upper area of her anterior trunk. She is embarrassed and very reticent to lift her blouse because her nipple will be exposed. How should the NP proceed? Examine all other areas of the trunk, then ask the child to lift her blouse When can a child with chickenpox return to daycare? After all lesions have crusted A topical treatment for basal cell carcinoma is: 5-fluorouracil. A 4-year-old has been diagnosed with measles. The nurse practitioner identifies Koplik’s spots. These are: found on the inside of the cheek and are granular. A child has 8-10 medium brown café au lait spots > 1 cm in diameter. The differential diagnosis should include: neurofibromatosis. An infant is diagnosed with diaper dermatitis. Satellite lesions are visible. This should be treated with a: topical antifungal agent. A pregnant mother in her first trimester has a 5- year-old who has Fifth Disease. What implication There is a risk of fetal death if she becomes does this have for the mother? infected. Impetigo is characterized by: honey-colored crusts. An example of a premalignant lesion that develops on sun-damaged skin is: actinic keratosis. The nurse practitioner is examining a 3-month-old infant who has normal development. She has identified an alopecic area at the occiput. What should be done? Encourage the caregiver to change the infant’s head position A patient with eczema asks for a recommendation for a skin preparation to help with xerosis. What should the NP respond? Use a petroleum-based product The term caput succedaneum refers to: scalp edema. A 9-year-old has been diagnosed with chickenpox. A drug that should be avoided in him is: aspirin. A 6-year-old patient with sore throat has coryza, hoarseness, and diarrhea. What is the likely etiology? Viral etiology A patient presents with small vesicles on the lateral edges of his fingers and intense itching. On close inspection, there are small vesicles on the palmar Dyshidrotic dermatitis surface of the hand. What is this called? The best way to evaluate jaundice associated with liver disease is to observe: the sclera, skin, and lips. The nurse practitioner identifies satellite lesions in a 6-month-old infant. These are: indicative of candidal infection. A child with a sandpaper-textured rash probably has: Strept infection. A young child has developed a circumferential lesion on her inner forearm. It is slightly raised, red and is pruritic. It is about 2.5 cm in diameter. This is probably related to: the child’s new cat. A 3-year-old female had a fever of 102° F for the last 3 days. Today she woke up from a nap and is afebrile. She has a maculopapular rash. Which statement is true? The rash will blanch. A “herald patch” is a hallmark finding in which condition? Pityriasis rosea The main difference between cellulitis and erysipelas is the: layer of skin involvement. A 15-year-old male has worked this summer as a lifeguard at a local swimming pool. He complains of itching in the groin area. He is diagnosed with tinea cruris. The nurse practitioner is likely to identify: well marginated half- moon macules on the inner thigh. The lesions seen in a patient with folliculitis might be filled with: pus. What advice should be given to a parent who has a child with Fifth Disease? A parent may experience joint aches and pains. A patient has used a high-potency topical steroid cream for years to treat psoriasis exacerbations when they occur. She presents today and states that this cream “just doesn’t work anymore.” What word describes this? Tachyphylaxis A child received a burn on his chest from a cup of hot coffee. On examination, the injured area appeared moist, red to ivory white in color, and features blisters. It is painful to touch. This burn would be classified as a: superficial partial thickness burn. A 6-year-old has been diagnosed with Lyme disease. Which drug should be used to treat him? Amoxicillin An adolescent has acne. The nurse practitioner prescribed a benzoyl peroxide product for him. What important teaching point should be given to Photosensitivity of the skin can occur this adolescent regarding the benzoyl peroxide? What is the proper technique to safely remove a tick from a human? Pull it off with tweezers What finding is most characteristic of shingles? Single dermatome affected A patient with diabetes has right anterior shin edema, erythema, warmth, and tenderness to touch. This developed over the past 3 days. There is no visible pus. What is the most likely diagnosis to consider? Cellulitis Endocrine Disorders A patient presents with consistently elevated blood glucose before his evening meal. What choice below represents an insulin change that would improve his evening glucose? Current AM regimen: 22u NPH, 12u short-acting insulin; Current PM regimen: 10u NPH, 8u short-acting insulin 24u NPH insulin in AM A patient has two fasting glucose values that were measured on two separate days in the same week. This patient: has impaired fasting glucose. What is the AM fasting glucose goal for a 75-year-old patient who has diabetes? 80-130 mg/dL Which medication used to treat diabetes is associated with diarrhea and flatulence? Metformin A patient presents with the following lab values. What is the diagnosis? Hypothyroidism A nurse practitioner has decided to initiate insulin in a 75-year-old patient who takes oral diabetic medications. How much long- acting insulin should be initiated in this patient, who weighs 100 kg? 5 units Which of the following patients is most likely to be diagnosed with hypothyroidism? 56-year-old female with swollen fingers A female patient has the following characteristics. Which one is NOT a risk factor for development Type 2 diabetes? Hypothyroidism A 38-year-old male patient, thought to be in good health, presents to a primary care clinic. On routine exam the patient’s fasting blood sugar is 242 mg/dL. A repeat value after eating is 288 mg/dL. Which of the following is least helpful in the initial Nonfasting lipids evaluation of this patient? A 65-year-old patient presents to your clinic with evidence of hyperthyroidism. In assessing her cardiovascular status, what should the NP assess immediately? Electrocardiogram A diabetic patient with albuminuria has been placed on an ACE inhibitor. How soon can the antiproteinuric effect of the ACE inhibitor be realized in this patient? 6-8 weeks The most appropriate screen for diabetic nephropathy is: urinary albumin to creatinine ratio and eGFR. A patient has been diagnosed with Grave’s disease. He is likely to have: an elevated T3 or T4. A 63-year-old male patient presents for evaluation of shortness of breath. His EKG demonstrates atrial fibrillation (a-fib). Which of the following laboratory studies would be LEAST helpful in the evaluation of the etiology of his symptoms? Fasting lipids Metformin is a good choice for many older adults with type 2 diabetes. However, while on metformin, the nurse practitioner should renal dysfunction. carefully monitor for: A 55-year-old female patient with diabetes has the following fasting lipid values and Hgb A1C. What is the relationship between Hgb A1C and this patient’s lipid values? As Hgb A1C decreases, triglycerides decrease. A 60-year-old female patient had an initial TSH result of 7 mIU/L (normal = 0.4- 3.8 mIU/L). Her repeat TSH and additional laboratory results are provided. What is the most likely diagnosis? Subclinical hypothyroidism A 28-year-old female has fatigue and weight loss of 4 pounds over the last 6 weeks. Her last menstrual period was 8 weeks ago. What is her most likely diagnosis? Hyperthyroidism What is the most sensitive laboratory assay for screening and identifying the vast majority of ambulatory patients with primary hypothyroidism? TSH only A female patient has the following characteristics. Which one represents the greatest risk factor for development of Type 2 diabetes? BMI 25 What is the earliest detectable glycemic abnormality in a patient with Type 2 diabetes? Postprandial glucose elevation A female patient has the following characteristics. Which one represents a risk factor for Type 2 diabetes? BMI 31 A patient has a TSH value of 13.1 today. The nurse practitioner has decided to initiate replacement with levothyroxine 88 mcg daily. When should the NP recheck the patient’s TSH level? 6 weeks A 30-year-old female patient who complains of fatigue undergoes a screening TSH. What should be done next? Repeat the TSH and add T4. A patient recently diagnosed with type 2 diabetes presents today with fever and burning with urination. She is diagnosed with a urinary tract infection (UTI). Which statement is correct? No specific conclusions can be drawn about the proteinuria. A 45-year-old patient who has hypothyroidism takes levothyroxine. Based on the following lab results, how should the nurse practitioner proceed? Adjust levothyroxine dose A 51-year-old overweight Caucasian male is diagnosed today with Type 2 diabetes. What medications should be initiated today? Metformin, atorvastatin, ramipril, ASA The most appropriate time to begin screening for renal nephropathy in a patient with Type 1 diabetes is: 5 years after diagnosis. A patient who is taking long-acting insulin basal insulin has elevated blood sugars. Which blood sugars are important to review in order to increase the dose of insulin? AM fasting Which laboratory abnormality commonly accompanies hypothyroidism? Dyslipidemia A 78-year-old was diagnosed with diabetes about 10 years ago. An older adult with a hypoglycemic episode is more likely to exhibit: dizziness and weakness. A normal fasting glucose in a nondiabetic patient is: less than 100 mg/dL. A patient who is diagnosed today with diabetes has an elevated urinary albumin to creatinine ratio. What can be concluded about this finding? The patient should have a repeat test in 3- 6 months. Hyperthyroidism may affect the blood pressure: by producing an increase in systolic and diastolic readings. A 69-year-old adult with coronary artery disease is found to have hypothyroidism. Which dose of levothyroxine is considered appropriate for initial treatment? 25 micrograms Mr. Smith, an overweight 48-year-old male with undiagnosed Type 2 diabetes mellitus, presents to your clinic. Which symptom is least likely associated with Type 2 diabetes mellitus? Constipation Mr. Jones, a patient with Type 2 diabetes, brings his obese 15-year-old son in to see the nurse practitioner. You examine the 15- year-old son and identify acanthosis nigricans. This probably indicates: insulin resistance. A patient brings in the following home glucose log. This patient: has normal blood glucose values. A patient who is 73 years old was diagnosed with diabetes several years ago. His A1C has remained elevated on oral agents and a decision to use insulin has been made. What is the goal postprandial glucose for him? Less than 180 mg/dL A diagnosis of Type 2 diabetes mellitus can be made: following fasting glucose values of 126 and 130 mg/dL on different days. A pregnant patient took levothyroxine prior to becoming pregnant. What should be done about the levothyroxine now that she is pregnant? She should continue it and have monthly TSH levels. A patient has nonfasting glucose values of 110 mg/dL and 116 mg/dL. This patient: has normal glucose values. A patient with a past history of treatment for hyperthyroidism is most likely to exhibit: hypothyroidism. Undiagnosed diabetes may present as: recurrent vaginal candidiasis. Which medication listed below is NOT associated with weight gain? Metoprolol A female patient has the following characteristics. Which one represents a risk factor for Type 2 diabetes? History of gestational diabetes A patient who has been treated for hypothyroidism presents for her annual exam. She complains of weight gain and fatigue. How should the NP proceed? Ask if she is taking her medication When the serum free T4 concentration falls: the TSH rises. A 45-year-old female patient has fatigue for the past 3 months and a 10-pound weight gain. She previously had regular menses occurring about every 30 days, but in the last 3 months her menses has varied between 30 and 45 days. What explains this finding? Hypothyroidism Which choice best describes the most common presentation of a patient with Type 2 diabetes? Insidious onset of hyperglycemia with weight gain A patient has two fasting glucose values (121 mg/dL and 136 mg/dL) that were measured on 2 separate days in the same week. This patient: should have a Hgb A1C performed. A 38-year-old male patient presents for his annual exam. He reports nervousness and weight loss, but denies any change in his dietary intake or exercise level. Based on these findings and the following lab values, what is his most likely diagnosis? Hyperthyroidism In order to determine how much T4 replacement a patient needs to reestablish a euthyroid state, the nurse practitioner considers: the patient’s body weight. A patient who has diabetes is taking pioglitazone (Actos). The nurse practitioner must remember to: order liver function studies in about 2-3 months. A 52-year-old presents with thirst and frequent urination today. His glucose is 352 mg/dL. How should this be managed today? Start insulin An 80-year-old patient who is overweight and sedentary has developed elevated fasting glucose levels (142, 153, and 147 mg/dL). She was diagnosed with diabetes today. Considering her age, how should the nurse practitioner proceed? Start metformin A patient has been diagnosed today with Type 2 diabetes. A criterion for diagnosis is: a fasting glucose > or equal to 126 and confirmed on a previous day. A 40-year-old patient with newly diagnosed Type 2 diabetes asks what his target blood pressure should be. The most correct response in mm Hg is: less than 140/90. A 50-year-old female presents for an annual exam. She complains of fatigue and weight gain. She has the following lab results. What should the NP order next? Repeat TSH plus free T4 A patient with hypothyroidism has been in a euthyroid state for several years. On screening, her TSH is elevated. The most likely cause of this is: substitution of levothyroxine for a generic medication. A 76-year-old obese patient has fatigue, thirst, and frequent urination. She was asked to measure AM fasting glucose values for 1 week. The values rage from 142-175 mg/dL. This patient: can be diagnosed with Type 2 diabetes. A 58-year-old female patient complains of fatigue and weight gain. Which laboratory study would be least helpful in determining the etiology of these symptoms? Sedimentation rate A 65-year-old diabetic has been on oral antihyperglycemic agents and is still having poor glycemic control. His AM fasting glucoses range from 140-160 mg/dL. You decide to add insulin. He weighs 127 kilograms. What should the NP order as an initial starting dose? 10 units long-acting insulin at bedtime The most appropriate time to begin screening for renal nephropathy in a patient with Type 2 diabetes is: at diagnosis. ENT Disorders What clinical finding necessitates an urgent referral of the patient to an emergency department? A fiery red epiglottis A 58-year-old farmer presents with a wedge- shaped, pinkish, clear growth on the nasal side of his eye. He states that it has been present for a while, but only recently began to feel as if a foreign body was in his eye. This is probably a: pterygium. A contact lens wearer presents with an erythematous right conjunctiva. He denies blurred vision. There is scant drainage and crusting around the right eyelashes. He reports that His visual acuity should be measured in each eye. crusting was present when he woke up this morning. How should the exam begin? A patient who presents with a complaint of sudden decreased visual acuity has a pupil that is about 4 mm, fixed. The affected eye is red. What might be the etiology? Glaucoma A patient is diagnosed with thrush. What might be found on microscopic exam? Budding yeasts, pseudohypha Which of the following is most likely observed in a patient with allergic rhinitis? Exacerbation of symptoms after exposure to an allergen An elderly patient who has a red eye with tearing was diagnosed with conjunctivitis. What characteristics below most closely indicate viral conjunctivitis? Profuse tearing A patient reports a penicillin allergy. What question regarding the allergy should the nurse practitioner ask to determine whether a cephalosporin can be safely prescribed? What kind of reaction did you have? A patient with a bacterial sinusitis cannot spread this to others via: urine or stool. A patient presents with severe toothache. She reports sensitivity to heat and cold. There is visible pus around the painful area. What is this termed? Pulpitis A 93-year-old demented adult has been recently treated for an upper respiratory infection (URI) but drainage from the right nostril persists. What should the NP suspect? Presence of a foreign body A patient stated that his ears felt stopped up. He pinched his nose and blew through it forcefully. The nurse practitioner diagnosed a ruptured left tympanic membrane. What would indicate this? Bright red blood in the left external canal What symptom tetrad is most commonly associated with infectious mononucleosis? Fatigue, fever, lymphadenopathy, pharyngitis A 70-year-old patient in good health has a large, white plaque on the oral mucosa of the inner cheek. There is no pain associated with this. What is a likely diagnosis? Leukoplakia On routine exam, a 15-year-old patient's tympanic membrane (TM) reveals a tiny white oblong mark just inferior to the umbo on the surface of the TM. The patient has no complaints of ear pain and gross hearing is intact. What is this? Scarring of the tympanic membrane A patient who is 65 years old states that she has “hay fever” and has had this since childhood. What agent could be safely used to help with rhinitis, sneezing, pruritis, and congestion? Nasal steroid On examination of the tympanic membrane (TM), a cloudy, red, bulging ear drum with impaired mobility is detected. This is consistent with: acute purulent otitis media. Group A Strept pharyngitis: can be accompanied by abdominal pain. Acute otitis media can be diagnosed by identifying which otic characteristic(s)? Cloudy, bulging TM with impaired mobility A 70-year-old patient has begun to have hearing loss. She relates that her elderly parents had difficulty hearing. Which complaint below is typical of presbycusis? Inability to hear consonants Which statement about serous otitis media is correct? This can be diagnosed with pneumatic otoscopy. A patient presents with findings of pain, warmth, redness, and swelling below the inner canthus toward nose. Tearing is present and when pressure is applied to the lacrimal sac, a purulent discharge from the puncta is noted. This is dacryocystitis. suggestive of: A teenager with fever and pharyngitis has a negative rapid strep test. After 24 hours, the throat culture reveals “normal flora." Which conclusion can be made? The pharyngitis is of undetermined etiology. Most commonly, epistaxis occurs: at Kiesselbach’s plexus. A patient has a penicillin allergy. He describes an anaphylactic reaction. Which medication class should be specifically avoided in him? Cephalosporins When examining vessels of the eye, the: arteries are smaller than the veins. A medication considered first line for a patient with allergic rhinitis is a: topical nasal steroid. A patient has been taking amoxicillin for 8 days for sore throat. Today, the patient has developed a pruritic full body rash and is diagnosed with penicillin allergy. What describes the skin manifestations of penicillin allergy? There will be hives. How should the class effect of the nasal steroids be described? There are no significant systemic effects with these. A patient with environmental allergies presents to your clinic. She takes an oral antihistamine every 24 hours. What is the most effective single maintenance medication for allergic rhinitis? Intranasal glucocorticoids A 39-year-old has a sudden onset of a painful right red eye. He reports sensitivity to light and the sensation of a foreign body, though his history for a foreign body is negative. He does not wear contact lenses. How should the NP manage this? Refer to ophthalmology A patient diagnosed with Strep throat received a prescription for azithromycin. She has not improved in 48 hours. What course of action is acceptable? A penicillin or cephalosporin with beta lactamase coverage should be considered. A patient with diarrhea has a positive enzyme immunoassay for C. difficile. He is on clindamycin for a tooth abscess. How should he be managed? Stop the clindamycin if possible, give metronidazole A patient who is otherwise healthy states that he woke up this morning and has been unable to hear out of his left ear. The Weber and Rinne tests were performed. What is the primary reason for doing this? It helps differentiate conductive from sensorineural hearing loss. A 17-year-old has a complaint of ear pain. If he has otitis externa, which complaint is most likely? Tragal pain A common complaint in older patients who have cataracts is: sensitivity to sunlight. A patient presents to clinic with a complaint of a red eye. Which assessment below rules out the most worrisome diagnoses? Usual visual acuity A 45-year-old patient describes a spinning sensation that has occurred intermittently for the past 24 hours. It is precipitated by position changes like rolling over in bed. During these episodes, he complains of intense nausea. Which choice best describes benign paroxysmal positional vertigo? Symptoms precipitated by a position change A 70 year-old male has a yellowish, triangular nodule near the iris. This is probably: a pinguecula. Which of the following symptoms is more indicative of a bacterial sinusitis than viral? Worsening of symptoms after initial improvement A 30-year-old male has been diagnosed with nonallergic rhinitis. Which finding is more likely in nonallergic rhinitis than allergic rhinitis? Older age of symptom onset An 80-year-old is having difficulty hearing. When the nurse practitioner examines him, she is unable to visualize the tympanic membrane because of cerumen impaction. This produces what kind of Conductive hearing loss? Arcus senilis is described as: normal in people > 50 years of age. In a patient who is diagnosed with mastoiditis, which of the following is most likely? Displaced pinna The single most effective maintenance therapy for allergic rhinitis is: a topical nasal steroid. A patient with allergic rhinitis developed a sinus infection 10 days ago. He takes fexofenadine daily. What should be done with the fexofenadine? Continue the fexofenadine and prescribe an antibiotic. A 61-year-old male presents with a 12-hour history of an extremely painful left red eye. The patient complains of blurred vision, haloes around lights, and vomiting. It began yesterday evening. On examination, the eye is red, tender and inflamed. The cornea is hazy and pupil reacts poorly to light. The most likely diagnosis in this patient is: acute angle glaucoma. A patient with mononucleosis has pharyngitis, fever, and lymphadenopathy. His symptoms started 3 days ago. What is a likely finding in this patient? He could have a negative Monospot. A 70-year-old presents urgently to the nurse practitioner clinic with angioedema. This began less than an hour ago. He is breathing without difficulty. What medication may have caused this? Fosinopril A 70-year-old female states that she sees objects better by looking at them with her peripheral vision. She is examined and found to have a loss of central vision, normal peripheral vision, and a normal lens. This best characterizes: macular degeneration. A nurse practitioner performs a fundoscopic exam. He identifies small areas of dull, yellowish- white coloration in the retina. What might these be? Cotton wool spots Papilledema is noted in a patient with a headache. What is the importance of papilledema in this patient? It could be an important finding in this patient. A patient presents to a nurse practitioner clinic with paroxysmal sneezing, clear rhinorrhea, nasal congestion, and facial pain. Which symptom below is NOT associated with allergic rhinitis? Facial pain A patient who is 52 years old presents to your clinic for an exam. You notice a yellowish plaque on her upper eyelid. It is painless. What should the NP assess? Lipid levels AV nicking may be identified in a patient who has what disease? Hypertension A patient presents with tragal pain. What is the most likely diagnosis? Otitis externa Mr. O has been diagnosed with hearing loss secondary to exposure to an ototoxic medication. Which one may be associated with ototoxicity? Aspirin The hearing loss associated with aging involves: sensorineural hearing loss. Which antihistamine is preferred for treating allergic rhinitis in an adolescent or adult? Once-daily, nonsedating A patient is diagnosed with otitis externa. He complains of tragal pain, otic discharge, otic itching, and fever. What is the cardinal symptom of otitis externa? Tragal pain A patient has nasal septal erosion with minor, continuous bleeding. There is macerated tissue. What is a likely etiology? Cocaine abuse A patient describes a sensation that "there is a lump in his throat." He denies throat pain. On exam of the throat and neck, there are no abnormalities identified. What is the most likely Globus reason this occurs? What medication should always be avoided in patients with mononucleosis? Amoxicillin A patient with mononucleosis would most likely have: lymphocytosis. A 4-month-old infant has thrush. The mother is breastfeeding. She reports that her nipples have become red, irritated, and sensitive. What should the nurse practitioner advise the mother of this baby to treat thrush? Treat the infant with an oral antifungal suspension and the mother’s nipples with a topical antifungal agent A patient has two palpable, tender, left preauricular nodes that are about 0.5 cm in diameter. What condition might this be associated with? Conjunctivitis Which long-acting antihistamine listed below is sedating? Cetirizine What are the most common signs and symptoms associated with mononucleosis? Fatigue and lymphadenopathy A 2-month-old is diagnosed with thrush. An exam of this patient’s saliva demonstrates all except: spores. A 12-year-old complains of itching in his right ear and pain when the pinna is pulled or the tragus is pushed. Examination revealed slight redness in the ear canal with a clear odorless fluid. This could be suggestive of: otitis externa. An older adult has a common cold. She calls your office to ask for advice for an agent to help her runny nose and congestion. What agent is safe to use? Guaifenesin Swimmer’s ear is diagnosed in a patient who has right-sided tragal tenderness. What other symptom might he have? Otic itching A 3-year-old has been diagnosed with acute otitis media. She is penicillin allergic (Type 1 hypersensitivity reaction). How should she be managed? Clarithromycin The patient presents with complaints of morning eyelash crusting and itchy red eyes. It began on the left and now has become bilateral. Based on the most likely diagnosis, what should the nurse practitioner tell the caregivers about this condition? This usually begins as a viral infection. A 3-year-old has been recently treated for an upper respiratory infection (URI) but drainage Presence of a foreign body from the right nostril persists. What should the NP suspect? A 20-year-old-patient presents with complaints of a sore throat and cough that began this morning. His oral temperature is 101.2 °F. He has the following laboratory results. What is his most likely diagnosis? Viral pharyngitis At what age would it be unusual to see thrush? At birth A rapid strep test was negative was negative in a 12-year-old female with tonsillar exudate, fever, and sore throat. Which statement is true regarding this? A throat swab should be collected and sent to microbiology. A patient has been diagnosed with acute rhinosinusitis. Symptoms began 3 days ago. Based on the most likely etiology, how should this patient be managed? Decongestant and analgesic An infant is brought to the nurse practitioner because his gaze is asymmetrical. Which finding indicates a need for referral to ophthalmology? He has persistent strabismus. The most common cause of acute pharyngitis in children is: respiratory viruses. Epstein-Barr virus is responsible for: mononucleosis. Which of the following will decrease the risk of acute otitis media in a 6-month-old? Breastfeeding A 6-day-old has bilateral mucopurulent eye discharge. Which historical finding explains the etiology of the discharge? Mother has chlamydia The nurse practitioner performs a fundoscopic exam on a patient who has recently been diagnosed with hypertension. What is the significance of AV nicking? This is indicative of long standing hypertension. A 4-year-old was diagnosed and treated for acute otitis media in the left ear 4 weeks ago. She is here today for a well-child visit. There is an effusion in the left ear. She denies complaints. How should this be managed? This should be monitored. A tympanic membrane (TM) is erythematous. Which factor listed below is NOT the cause of an erythematous TM? Coughing A patient has been diagnosed with mononucleosis. Which statement is correct? Cervical lymphadenopathy may be prominent. A 6-month-old infant has a disconjugate gaze. The The infant will have an nurse practitioner observes that the 6-month-old tilts his head when looking at objects in the room. Which statement is true? abnormal cover/uncover test. When does a child’s vision approximate 20/20? 5-6 years An otherwise healthy 6-year-old male has been diagnosed with otitis media. His mother reports that he has not had an ear infection since he was 3 years old. How long should he be treated with an antibiotic? 5-7 days An older adult has cerumen impaction in both ears. His hearing is diminished. This type of hearing loss is: conductive. A 3-year-old has fluid in the middle ear that does not appear infected. The eardrum appears normal. This is referred to as: middle ear effusion. At what age should oral health risk assessment begin? 6 months A 14-year-old was diagnosed and treated for left acute otitis media 4 weeks ago. She presents today for a follow-up visit. There is an effusion in the left ear. She denies complaints. How should this be managed? This should be monitored. A patient presents to your clinic with a painless red eye. Her vision is normal, but her sclera has a blood red area. What is this termed? Subconjunctival hemorrhage What is the usual age for vision screening in young children? 3 years An NP examines a screaming 2-year-old. A common finding is: pink tympanic membranes. A 4-year-old child with otitis media with effusion: probably has just had acute otitis media. In a patient with mononucleosis, which laboratory abnormality is most common? Lymphocytosis and atypical lymphocytes The most common complication of influenza is: bacterial pneumonia. bacterial pneumonia. A 2-year-old has a sudden onset of high fever while at Treat the otitis daycare. The daycare attendant describes a seizure in the media and give child. The child is brought to the clinic; neurologically he education about appears normal. His body temperature is 99.9° F after fever management receiving ibuprofen. He is diagnosed with otitis media. How should the nurse practitioner manage this? A 32-year-old patient is a newly diagnosed diabetic. She has developed a sinus infection. Her symptoms have persisted for 10 days. Six weeks ago, she was treated with amoxicillin for an upper respiratory infection. It cleared without incident. What should be recommended today? Prescribe amoxicillin- clavulanate today. Gastrointestinal Disorders An 85-year-old adult has chronic constipation. What is the most likely cause of her constipation? Medication-related Choose the number on the image that most closely approximates the location of an indirect inguinal hernia. 4 A patient has received a prescriptions for metronidazole for treatment of C. difficile. What should be avoided in this patient? Alcohol An inguinal hernia is palpated on a male patient by an examiner. Which word below best describes what the hernia feels like when touched by the examiner? Silky The relationship between duodenal ulcer disease and H. pylori infection is: very likely. The two tests that can indicate current infection with hepatitis B are: presence of hepatitis B surface antigen and IgM. A 31-year-old patient is suspected of having hepatitis C. He reports possible exposure about a month ago. How should the nurse practitioner interpret his laboratory results? The patient does not have hepatitis C A 79-year-old with an appendicitis is unlikely to exhibit: initial WBC elevation. A patient has been diagnosed with hepatitis A. The most common reported risk factor is: international travel. What medication used to treat patients who have GERD provides the fastest relief of heartburn symptoms? Calcium carbonate Most patients who have acute hepatitis B infection: have varied clinical presentations. A patient has the following laboratory values. What does this mean? He has acute hepatitis B. A patient presents with complaints of bright red stools over the past week. This symptom could be consistent with: cancer of the sigmoid colon. A 19-year-old female presents with lower abdominal pain that began about 12 hours ago. She denies vaginal discharge. Which choice below is the least likely cause of her symptoms? Renal stone A 15-year-old is about 10% below her ideal body weight. Laboratory studies were performed. Which complaint might be common in this patient? Dizziness with standing A patient with gall bladder disease has classic symptoms. Which symptom below is NOT classic of gallbladder disease? Pain that occurs when the stomach empties GERD (gastroesophageal reflux disease) and physiologic reflux have similar characteristics. However, physiologic reflux: rarely occurs at nighttime. A 35-year-old patient has the following laboratory values. How should they be interpreted? The patient should consider Hepatitis B immunization. Choose the number on the image that most closely approximates the location of a direct inguinal hernia. 1 An 83-year-old patient is diagnosed with diverticulitis. Where is her pain typically located? Left lower quadrant A patient has the following lab values. This indicates that he: is immune to hepatitis B. A patient has suspected peptic ulcer disease. Her symptoms occur a few hours after eating. She probably has a(n): duodenal ulcer. The relationship between colon polyps and colon cancer is that polyps: have a slow progression to colon cancer. Which of the following would be usual in a patient with biliary colic? Pain in upper abdomen in response to eating fatty foods Which term does not characterize hemorrhoids? Carcinogenic A 26-year-old female presents with concerns about possible hepatitis C (HCV) infection. She admits to IV drug use 2 months ago and sharing needles with several other people. Initial laboratory studies have been completed. How should this be managed? Order HCV RNA A patient has the following lab results. This means: more data is needed. Most patients who have acute hepatitis A infection: have a self-limited illness. What choice below is most commonly associated with pancreatitis? Gallstones and alcohol abuse A 20-year-old female patient presents with tenderness at McBurney’s point. Appendicitis is considered. What laboratory test should be done initially to determine the etiology of this patient's abdominal pain? Serum pregnancy test A 70-year-old presents to the nurse practitioner’s office for a well exam today. What medication probably has no effect on screening for occult blood in the stool? Acetaminophen A 5-year-old has been diagnosed with pinworms. He lives with his mother. There are no other members of the household. How should his mother be managed? Perform the “scotch tape” test and look at the collection under the microscope. Which of the following is NOT a risk factor for developing hepatitis B? Consuming contaminated water A 50-year-old with a history of consumption of 3- 4 alcoholic drinks daily and weekend binges has elevated liver enzymes. Which set of enzymes is most representative of this patient? AST= 200, ALT= 75 A patient has had right upper quadrant pain that has lasted for the past 3 days, but the pain has Serum amylase become acute in the past 12 hours. He has low- grade fever. Which lab test(s) will be elevated if he has pancreatitis? A 40-year-old patient has the following laboratory values. How should they be interpreted? The patient has been immunized. Older adults frequently complain of constipation. Which medication listed below does NOT increase the likelihood of constipation in an older adult? Metformin Many older adults have cachexia. What characterizes this? Illness and loss of muscle mass What medication may be used to treat GERD if a patient has tried over-the-counter ranitidine without benefit? Pantoprazole Which of the following symptoms is typical of GERD? Pyrosis The most common place for indirect inguinal hernias to develop is: the internal inguinal ring. A 42-year-old patient was diagnosed with ulcerative colitis many years ago. What part of He should have a colonoscopy every 1-5 his routine health screenings should be stressed by the nurse practitioner? years. A 37-year-old has routine blood work performed during an annual exam. On exam he has a tender, enlarged liver. How should the nurse practitioner proceed? Order a hepatitis panel. An 85-year-old adult has chronic constipation. How should this be managed initially? Avoid all constipating medications/foods when possible The three most common causes of bacterial diarrhea in the US are Salmonella, Campylobacter, and: Shigella. A 45-year-old patient has the following laboratory values. How should they be interpreted? The patient has hepatitis. A patient has the following laboratory values. What does this mean? He has no immunity to hepatitis A. The most common reason that older adults develop peptic ulcer disease is: H. pylori infection. Which set of symptoms is most likely in a patient infected with C. difficile? Diarrhea, abdominal pain, nausea or vomiting A 56-year-old male patient has been diagnosed with an inguinal hernia. What symptom would make the nurse practitioner suspect an incarcerated hernia? Pain Which choice contains three medications that should have liver function tests measured prior to initiation of the medications? Terbinafine, atorvastatin, simvastatin A healthcare provider (“the HCP”) was stuck with a needle from a patient suspected to be infected with HIV (“the patient”). A rapid HIV test was performed on the patient and found to be positive. This means that: the HIV status of the patient requires further testing. A patient with diarrhea is tested for C. difficile. How soon should the enzyme immunoassay (EIA) yield results? About 24 hours An 83-year-old patient is diagnosed with diverticulitis. The most common complaint is: left lower quadrant pain. Which medication listed below can exacerbate the symptoms of GERD? Verapamil A 24-year-old female presents with pain and tenderness in the right lower abdominal quadrant. Her pelvic exam and urinalysis are within normal limits. Her WBC is elevated and Appendicitis her urine pregnancy test is negative. What is part of the differential diagnosis? What medication listed below could be used to increase appetite in an anorexic patient? Megestrol Which symptom is INCONSISTENT with irritable bowel syndrome in older adults? Onset after 50 years of age The early signs and symptoms of appendicitis in an adult: are subtle. A 24-year-old male has recently returned from a weekend camping trip with friends. He has ulcerative colitis and history of migraine headaches. He reports a 2-day history of headache, nausea, and vomiting with weakness. Which of the following is not part of the differential diagnosis? Exacerbation of ulcerative colitis The most common symptoms associated with gastroesophageal reflux disease (GERD) are heartburn and: regurgitation and dysphagia. A patient with a suspected inguinal hernia should be examined: standing. A 26-year-old female complains of pain at order a CBC and McBurney’s point. She feels nauseated. Her vital signs are provided. The most appropriate initial action by the NP is to: pregnancy test. A 95-year-old male has lost muscle mass as he has aged. He does not have any underlying disease that has caused this loss. What is this termed? Sarcopenia A patient has been diagnosed with viral gastroenteritis. He has nausea and vomiting, and has started having lower abdominal cramps. What is the most effective intervention for him? Oral rehydration A 20-year-old female patient presents with tenderness at McBurney’s point. What laboratory test supports a diagnosis of appendicitis? CBC with elevated white count A 43-year-old female patient reports a possible exposure to hepatitis C about 4 months ago. Which statement is true about this patient? The patient does not have hepatitis C, but has immunity to hepatitis B. What is the simplest screen for nutritional adequacy in elderly patients? Measure their weight An 82-year-old adult has constipation. A supplement known to cause constipation is: calcium. A patient asks for advice about a medication that will produce rapid relief if he is having heartburn symptoms. What should the nurse practitioner recommend? Calcium carbonate A fecal occult blood test (FOBT) obtained during a rectal examination: is inadequate to screen for colorectal cancer. Which is NOT an effective strategy for helping older adults gain weight? Increase carbohydrate intake A 70-year-old patient states that he had some bright red blood on the toilet tissue this morning after a bowel movement. He denies pain. What is the LEAST likely cause in this patient? Anal fissure A patient has elevated liver enzymes. What is the likely etiology of the elevation? Hepatitis Which description is more typical of a patient with acute cholecystitis? The patient is ill- appearing and febrile. Which imaging study of the abdomen would be LEAST helpful in diagnosing an acute appendicitis? X-ray An older patient presents with left lower quadrant pain. If diverticulitis is suspected, how CT scan of abdomen should the NP proceed? Children with an inguinal hernia: have a history of an intermittent bulge in the groin. An 84-year-old presents with a stated involuntary weight loss. He states that he’s lost about 6 pounds in the last 6 or 8 weeks. What statement below is NOT part of the assessment? Evaluate his upper and lower extremity muscle mass. The most common cause of diarrhea in adults is: viral gastroenteritis. What is true regarding older adults who are overweight? Mortality in older adults related to overweight states declines over time. A 3-day-old full-term infant has a bilirubin level of 16 mg/dL. How should this be managed? Order phototherapy for the infant A patient has hepatitis B. He probably has a predominance of: lymphocytes. An older adult has suspected vitamin B12 deficiency. Which of the following lab indices is more indicative of a vitamin B12 deficiency? Macrocytosis A 40-year-old patient has the following laboratory values. How should they be interpreted? The patient had hepatitis. A patient is in the clinic with a 36-hour history of diarrhea and moderate dehydration. Interventions should include: oral rehydration with an electrolyte replenishment solution. A patient has the following laboratory value. What is the clinical interpretation? He has immunity to hepatitis A. A 7-year-old male presents with encopresis. The NP might expect: constipation. Mrs. Lovely, an 84-year-old, complains of fecal incontinence. A likely cause is: constipation. A patient presents with right upper quadrant and upper abdominal pain. Acute cholecystitis is suspected because the pain radiates to the: right scapula area. The initial step in the management of encopresis is: client and family education. Hirschsprung's disease is characterized by: failure to pass meconium in the first 48 hours of life. A mother of a 4-week-old infant visits your office. She states that her baby is vomiting after feeding and then cries as if he is hungry again. What should the nurse practitioner assess? His abdomen for an olive shaped mass A 14 year-old male patient has an acute, painless groin swelling. Which tool would yield the most information to identify the etiology of the swelling? Ultrasound of the scrotum A mother presents with her 1-month-old infant. She reports that he cries inconsolably every evening after his first evening feeding. She asks for help. What should be done? Provide education, parental reassurance, and encouragement. Symptoms of uncomplicated reflux disease in older adults should be treated: with empiric treatment. A 6-week-old male infant is brought to the nurse practitioner because of vomiting. The mother describes vomiting after feeding and feeling a “knot” in his abdomen especially after he vomits. The child appears adequately nourished. What is the likely etiology? Pyloric stenosis A 5-year-old female’s playmate has been diagnosed with pinworms. The mother brings her child in for an exam. The 5 year-old denies rectal itching. How should the NP proceed? Perform the “scotch tape” test and look at the collection under the microscope. Which patient has the least worrisome symptoms associated with his diarrhea? One with: moderate amounts of watery diarrhea. A 48-year-old patient has the following laboratory values. How should they be interpreted? The patient has hepatitis C. A patient has been diagnosed with hepatitis B. The most common reported risk factor is: sexual exposure. A 63-year-old male has been your patient for several years. He is a former smoker who takes simvastatin, ramipril, and an aspirin daily. His blood pressure and lipids are well controlled. He presents to your clinic with complaints of fatigue and “just not feeling well” for the last few days. His vital signs and exam are normal, but his liver enzymes are elevated. His hepatitis panel is negative for infectious hepatitis. What is the most likely cause of his elevated liver enzymes? Daily grapefruit consumption for the past 10 days Which of the following is an appropriate initial intervention for gastroesophageal reflux disease (GERD) in an 8-week-old? Small, frequent thickened feedings A 10-year-old female presents with a 3-month history of abdominal pain. She has been diagnosed with recurrent abdominal pain. During the interview the nurse practitioner is likely to elicit a school absenteeism. finding of: A patient has a positive hepatitis B surface antibody. This means he: is immune to hepatitis B. Health Promotion Disorders Screening for abdominal aortic aneurysm should take place: once for all males aged 65-75 who have ever smoked. A contraindication to giving MMR vaccination is: encephalopathy within 7 days after immunization. An older adult has osteopenia. Her healthcare provider has recommended calcium 500 mg three times daily. What is the most common side effect of calcium supplementation? Constipation What choice below would be beneficial to a 76-year- old who takes daily oral steroids for COPD and now takes a daily aspirin for primary prevention of myocardial infarction? Daily proton pump inhibitor (PPI) In older adult females, which screening test has demonstrated greatest reduction in mortality from cancer? Breast A 20-year-old student has an MMR titer that demonstrates an unprotective titer for rubella. She is HIV positive. Her CD4 cell count is unknown. Which statement is true? She is at risk for MMR but should not be immunized. At what age should hearing screening take place in older adults according to Assessing Care in Vulnerable Elders (ACOVE-2)? 75 years What is the most common nutrition syndrome in older adults? Undernutrition A 76-year-old patient who is very active has elevated cholesterol and LDLs. He had been treated for hypertension for > 10 years with near normal blood pressures. What is the current recommendation for managing his lipids? He should be treated with a statin. The incidence of osteoporosis in older adults is high. Which characteristics below would increase the risk of osteoporosis in an older adult male patient? Chronic glucocorticoid therapy, age 70 years Which finding below is considered “within normal limits”? An INR of 2.0 in a patient taking warfarin A mammogram in a healthy 50-year-old female patient is an example of: secondary prevention. A 74-year-old who retired as a store clerk last year would like to start an exercise program. She tells the nurse practitioner that she would like to start walking 15-20 minutes daily. Which statement is correct regarding the patient? She does not need any testing because she is asymptomatic. What is the recommendation from American Cancer Society for assessment of the prostate gland in a man who is 45 years old and of average risk for development of prostate cancer? He should have: screening starting at 50 years of age. An octogenarian asks the nurse practitioner if it is OK for him to have an alcoholic beverage in the evenings. There is no obvious contraindication. How should the nurse practitioner respond? Yes, but not more than 1- 2 drinks per day. A side effect of DTaP that should be reported is: nonstop crying (3h or more) within 48 hours of receiving the immunization. A 7-year-old entered the clinic 1 month ago. There was no evidence that he had any immunizations. He was given the vaccinations listed on his vaccination record at the time of his visit. If he returns today, which immunizations can he receive? Hepatitis B, DTaP, IPV, MMR Which pharmacokinetic factor is influenced by a decrease in liver mass in an older adult? Metabolism A criterion for medication choice in an older adult is: half-life less than 24 hours. A patient is 86 years old and functions independently. He has hypertension, hyperlipidemia, BPH, and flare- ups of gout. His last colonoscopy was at age 76 years. What should he be advised about having a colonoscopy? It is not advised in this patient at this time What should the nurse practitioner recommend to any elder taking medications? Keep a list of all of your medications with you A 75-year-old adult asks for the pneumonia vaccine. His immunization record indicates that he had one at age 65 and another a year later. What is the recommendation of the CDC about how the NP should handle his request? Do not revaccinate this patient at this time The average age of pubertal growth spurt in North American boys is: 12 - 14 years. A 58-year-old patient has an annual exam. A fecal occult blood test was used to screen for colon cancer. Three were ordered on separate days. The first test was positive; the last two were negative. How should the nurse practitioner proceed? Refer him for a colonoscopy. Risk assessment for dyslipidemia should begin at: 2 years. A 12-month-old is here today to receive the varicella immunization. A patient’s mother reports that her 12- month-old child was exposed to chickenpox about a week ago. The NP should recommend that he: be given the vaccine. Two common causes of weight loss in older adults are: depression and malignancy. The pneumococcal immunization in infants has: shifted the pathogenesis to fewer cases of S. pneumoniae. A 2-month-old infant has an asymmetric Moro reflex. Which statement is true? The infant could have a birth injury. Which reflex would NOT be expected in a 1-month- old? Parachute At what age should initial blood pressure screening take place? 3 years A patient who is 62 years old asks if she can get the shingles vaccine. She has never had shingles but states that she wants to make sure she doesn’t get it. What should the nurse practitioner advise? You are eligible to receive it but you still may get shingles. A patient who has been treated for hypothyroidism presents for her annual exam. Her TSH is 4.1 (normal = Repeat the TSH in 2-3 weeks. 0.4- 3.8). She feels well. How should she be managed? MMR immunization is safe in children: who are allergic to eggs. When does a child’s vision approximate 20/20? 5-6 years What would be age appropriate anticipatory guidance for the parent of a 9-month-old infant? Discuss weaning from a bottle. A 2-year-old with sickle cell anemia (SCA) should receive which immunizations? All routine childhood immunizations at the usual time A 6-month-old child comes into the clinic for immunizations. Which item below allows a delay in his getting immunizations today? Child has otitis media with temperature of 103° F Which of the following will decrease the risk of acute otitis media in a 6-month-old? Breastfeeding Babies up to one year of age are called: infants. A pregnant patient is concerned because her 12 month-old needs an MMR immunization. What should the NP advise this patient? MMR immunization presents no risk to the child’s mother. Immunize now. What is the recommendation for daily multivitamin supplementation in older adults? It has no proven benefit. A 13-year-old male has exhibited the first sign that he is experiencing sexual maturation. He has: an increase in testicular size. The chest circumference of a 12 month-old is: equal to head circumference. An oral antifungal agent is commonly used to treat tinea unguium. The difficulty in treating an older adult with this is infection is: tolerability of the medication. A 7-year-old enters the nurse practitioner clinic. There is no evidence that he has received any immunizations. What should be administered today? Hepatitis B, Tdap, IPV, varicella, MMR Which recommendation below reflects CDC’s recommendation for administration of the zoster vaccine? It should be given to immunocompetent adults age 60 and older. A young female has breast buds bilaterally. This represents Tanner Stage: II A 3-day-old infant weighed 8 pounds at birth. Today he weighs 7.5 pounds. How should this be managed? Continue feeding every 2- 4 hours What Tanner stage corresponds to an average 8-year- old male? Stage 1 Which reflexes might a 1-month-old infant be expected to exhibit? Moro, stepping, rooting What is the earliest age that an average child would appropriately receive construction paper and a pair of scissors with rounded points? 4 years A 67-year-old patient presents an immunization record that reflects having received the PCV13 immunization when she was 65 years old. She received the PPSV23 immunization one year later. Which statement below reflects the current standard of practice recommended by ACIP and CDC for this patient? She does not need the immunization. A 1-year-old patient’s mother reports allergy to gelatin. The mother describes the reaction as “lips swelling and breathing difficulties that necessitated a trip to the emergency department.” Which immunizations should be avoided? Varicella and MMR What is true regarding the shingles vaccine given to adults at or after age 50? It contains significantly more virus than the chickenpox vaccine. Which patient below is most likely to experience stranger anxiety during a physical exam? A 12-month-old female Head circumference should be measured until a child has attained: 36 months of age. Which reflex may be present at 9 months of age during sleep? Rooting What temperature should be set on a water heater in the home of an older adult to prevent burn injury? Less than 120 degrees A mother reports that her child is not allergic to chickens but is allergic to ducks and duck feathers. The child is 4 years old today. Which immunizations should he receive? None are contraindicated A 6-month-old infant has a disconjugate gaze. The nurse practitioner observes that the 6-month-old tilts his head when looking at objects in the room. Which statement is true? The infant will have an abnormal cover/uncover test. What is the usual age for vision screening in young children? 3 years When is a child first able to stand on one foot? 3 years What is the usual recommendation about administering MMR and varicella immunizations? They should be given on the same day or at least one month apart. The first sign that a male child is experiencing sexual maturation is: increase in testicular size. An NP examines a screaming 2-year-old. A common finding is: pink tympanic membranes. A patient who wrote a living will has changed his mind about the initiation of life-sustaining measures. Which statement is true about this? He can only change the content if he is of sound mind. The age at which a child can first walk backwards is: 18 months. An infant is brought to the nurse practitioner because his gaze is asymmetrical. Which finding indicates a need for referral to ophthalmology? He has persistent strabismus. A child who can stack a maximum of five blocks is probably: 2 years of age. The term caput succedaneum refers to: scalp edema. A child who is 15 months old is referred to as a(n): toddler. At what age should oral health risk assessment begin? 6 months A full-term newborn is diagnosed with hyperbilirubinemia. When would his bilirubin level be expected to peak? 3-4 days What is the earliest age that an average child would be able to copy a triangle, know his colors, and count on his fingers? 5 years Hematology Disorders A 66-year-old African American male complains of pain in his trunk, especially his ribs. Cardiovascular disease is ruled out. He has a normocytic, normochromic anemia with hypercalcemia. The differential diagnosis should include: multiple myeloma. A patient has heavy menses. Which lab value below reflects iron deficiency anemia? Increased TIBC A 65-year-old male is being treated with oral medications for hypertension, hyperlipidemia, diabetes, and osteoarthritis. The most likely reason for his osteoarthritis. iron deficiency anemia is oral treatment for: The following lab values most likely indicate what type of anemia? Vitamin B12 deficiency An African American male complains of pain in his back and trunk. He is diagnosed with multiple myeloma. He is probably: about 65 years old. A 70-year-old male who is a smoker has lymph nodes in his axillary and inguinal areas that are palpable but nontender. He states that he feels well today. However, he has had a nonproductive cough for the past 4 weeks. What should be included in a differential diagnosis for this patient? Lymphoma Besides inadequate intake of vitamin D in older adults, which other factor contributes to deficiencies? Impaired synthesis of previtamin D An 18-year-old female patient has iron deficiency anemia. If this anemia has occurred in the past 3-4 months, what might be expected? An increased RDW A 26-year-old female has thalassemia minor. What should be limited in her diet to avoid Multivitamin with iron hepatotoxicity? A definitive diagnosis of sickle cell anemia can be made: by a hemoglobin electrophoresis. Which statement is true about vitamin B12? Inadequate amounts can produce cognitive changes. A patient demonstrates leukocytosis. This means: he has an infection of unknown origin. A 75-year-old male patient has been in poor health for a decade. Which type of anemia is LEAST likely in this patient? Sideroblastic anemia Which anemias should be included in the differential based on this patient's CBC? Iron deficiency anemia and thalassemia A serum ferritin level: demonstrates the amount of iron in storage. A healthy appearing 3-year-old female child presents with lymphadenopathy, nonblanchable redness over both knees and elbows, and a low grade fever. During the exam, she is found to have normal growth acute lymphocytic leukemia. and development and interacts appropriately with the nurse practitioner. She had an upper respiratory infection about 4 weeks ago. A CBC and urinalysis were obtained. The most likely diagnosis is: A patient is being treated for iron deficiency anemia. Iron is better absorbed: on an empty stomach. What statement is true about anemia in older adults? Anemia may have more than one origin and co-exist in older adults. Leukemia may have varied clinical presentations. Which characteristic would be unusual to find in a patient with leukemia? Sickle shaped cells Thalassemia minor can be recognized by: microcytic, hypochromic red cells. A vitamin B12 deficiency might be suspected in an older patient with what complaints? Memory issues and glossitis Lead toxicity can be associated with: sideroblastic anemia. A 50-year-old female presents to your clinic with symptoms of fatigue over the past 1-2 months. Preliminary labs have been performed. What should the NP do next? Prescribe an iron supplement A patient is found to have anemia. What term is used to describe the patient's MCH in this CBC? Normochromic A patient has been treated for HIV infection with antiretroviral therapy. He is stable. How often should CD4 counts be repeated? Every 3-6 months A 34-year-old female has been recently diagnosed with rheumatoid arthritis and is being cared for by a rheumatologist. Her current medication list includes methotrexate, levothyroxine, and Depo- Provera contraceptive. She presents to your primary care clinic for her annual examination. What diagnosis is likely? Folic acid deficiency anemia A female patient has been diagnosed with Glucose-6-phosphate dehydrogenase deficiency (G6PD). What should be done to prevent lysis of red cells in this patient? Avoid aspirin and sulfa drugs A patient had a splenectomy after an automobile accident 3 months ago. Patients more susceptible to bacterial infection. who are asplenic are: An older adult has suspected B12 deficiency. Which of the following lab indices is most indicative of a B12 deficiency? Increased MCV A 12-month-old was screened for iron deficiency anemia and found to be anemic. The nurse practitioner ordered oral iron. In 1 month, the child’s hemoglobin was reassessed. It increased greatly. Which answer might account for this? The bone marrow is responding appropriately. A 2-year-old with sickle cell anemia (SCA) should receive which immunizations? All routine childhood immunizations at the usual time A patient with iron deficiency anemia takes iron supplementation daily. What should he be advised to avoid within a couple of hours of taking iron? An antacid A patient complains of numbness and tingling in both hands. He has the following lab values. The nurse practitioner should suspect: iron deficiency anemia. What diagnosis is likely based on this patient's laboratory values? Pernicious anemia A patient has the following CBC. The most likely interpretation is that he: has an infection of unknown etiology. The following lab values most likely indicate what type of anemia: thalassemia. An older adult has suspected B12 deficiency. Which of the following lab indices is more indicative of a B12 deficiency? Macrocytosis Which suggestion below is the standard for treating iron deficiency anemia in infants and children? Iron supplementation in divided doses between meals with orange juice An obese 78-year-old male has the following lab values. The etiology of his anemia is likely secondary to: presence of chronic disease. A measure of the degree of variation in red cell size is indicated by: RDW. A 73-year-old female is found to be anemic. Which diagnosis is likely based on this Vitamin B12 deficiency patient’s laboratory values? A 72-year-old male has been diagnosed with anemia. Which of the following statements is true? He may have a GI bleed. A patient has been diagnosed with HIV. The patient’s viral load was ordered. What other test may be ordered to assess the status of the patient’s immune system? CD4 cell count Which hallmark finding is associated with both B12 and folate deficiencies? Macrocytosis A patient with anemia of chronic disease probably has a: normocytic anemia. A child and father live in a house that was built in the 1960s. They both are found to be lead toxic. Which type of anemia is typically observed in patients who are lead toxic? Iron deficiency anemia The nurse practitioner sees a child who reports fatigue and presents with purpura on his lower extremities. His temperature is normal. The differential includes: acute leukemia. A patient with diarrhea has a stool specimen positive for WBCs. What is a likely etiology? A viral infection Which white cell should be present in the greatest number in a patient who is healthy today? Neutrophils A patient presents with a complaint of dark- colored urine and fatigue. What is the most likely explanation? Glomerulonephritis A patient with pernicious anemia may be observed to have: glossitis. What choice below can be attributed to the two most common causes of iron deficiency anemia in adults? Gynecologic losses and bleeding in the gastrointestinal tract A patient has had an allergic reaction to seafood with anaphylaxis 48 hours ago. Which white cell will probably be abnormal? Eosinophils A patient is found to have an anemia. What term is used to describe this patient's MCV? Normocytic Thrombocytopenia may present as: purpura. A B12 deficiency can produce: pernicious anemia. A 26 year old patient has the following CBC. An expected finding is: asthma exacerbation. A healthy appearing 3-year-old female presents with nonblanchable redness over both knees and elbows. During the exam, she is found to have normal growth and development, and she interacts appropriately with the nurse practitioner. She had an upper respiratory infection about 4 weeks ago that cleared without incident. A CBC and urinalysis were obtained. The most likely diagnosis is: idiopathic thrombocytopenia purpura (ITP). The following lab values most likely indicate what type of anemia? Iron deficiency anemia A 28-year-old female patient reports fatigue for the past 4-6 weeks. Based on her most likely diagnosis, she will receive an oral supplement as treatment for her anemia. For best absorption, it should be taken: on an empty stomach. Which disease listed below can be associated with lymphocytosis? Mononucleosis A female vegetarian presents to your clinic with symptoms of fatigue. What can the NP suggest she eat to help with resolution of iron deficiency anemia? Dark-green leafy vegetables and dried peas and beans The laboratory identifies metamyelocytes in a 50-year-old patient who had a CBC performed. What might be an expected finding in this patient? Splenomegaly Men’s Health Disorders What is the effect of digital rectal examination (DRE) on a male’s PSA (prostate-specific antigen) level if it is measured on the same day as DRE? The change is insignificant A localized tumor in the prostate gland associated with early-stage prostate cancer is likely to produce: no symptoms. Digital rectal exam (DRE) may be performed to assess the prostate gland. Which term does NOT describe a prostate gland that may have a tumor? Boggy A 70-year-old male presents to your clinic with a lump in his breast. How should this be evaluated? Mammogram and ultrasound A patient with testicular torsion will have a: negative cremasteric reflex on the affected side. Noninfectious epididymitis is common in: truck drivers. What is the American Cancer Society’s recommendation for prostate screening in a 70- year-old male? He should be screened until he has a life expectancy of less than 10 years. What class of medications can be used to treat benign prostatic hyperplasia and provide immediate relief? Alpha-1 blockers What is the recommendation of the American Cancer Society for initial screening of an African- American male for prostate cancer? Discussions starting at age 40-45 years A 25-year-old male patient is training for a marathon. He reports an acute onset of scrotal pain after a 10-mile run. He has nausea and is found to have an asymmetric, high-riding testis on the right side. What should be suspected? Testicular torsion What symptom listed below might be seen in a male patient with benign prostatic hyperplasia? Nocturia A 65-year-old Caucasian male has a firm, nontender, symmetrically enlarged prostate gland on examination. His PSA is 3.9 ng/mL. His PSA level is influenced by: race, age, and volume. Hematuria is NOT a common clinical manifestation in: early prostate cancer. What is the recommendation from the American Cancer Society (ACS) for screening for prostate cancer in an African American male who is 50 years old? PSA screening now if desired A 22-year-old male who is otherwise healthy complains of scrotal pain. His pain has developed over the past 4 days. He is diagnosed with epididymitis. What is the most likely etiology? Infection with Chlamydia What is the recommendation from the American Cancer Society (ACS) for assessment of the prostate gland in a man who is 45 years old and of average risk for development of prostate cancer? Consider PSA screening at 50 years of age A 28-year-old male patient has epididymitis. His most common complaint will be: scrotal pain. A 71-year-old male patient is taking dutasteride (Avodart), a 5-alpha-reductase inhibitor. What It will decrease. affect might this have on his PSA level? 5-alpha-reductase inhibitors work by producing: a decrease in the size of the prostate. Which of the following medications should be avoided in a 65-year-old male with benign prostatic hyperplasia (BPH)? Diphenhydramine (Benadryl) A 40-year-old male has been diagnosed with acute bacterial prostatitis. His prostate-specific antigen (PSA) is elevated on diagnosis. How soon should his PSA be rechecked? 4 weeks A common presentation of an inguinal hernia is: groin or abdominal pain with a scrotal mass. Hesselbach’s triangle forms the landmark for: inguinal hernia. The following PSA levels have been observed in a patient. What conclusion can be made following these annual readings? There is a steady increase that is worrisome. An 80-year-old white male presents for an annual exam. He complains of difficulty initiating his urinary stream but denies burning and urgency with urination. On exam, he is found to have a firm, smooth, nontender, and symmetrically enlarged Benign prostatic hypertrophy prostate. What diagnosis is consistent with these findings? Which of the following results in a clinically INSIGNIFICANT increase in the prostate-specific antigen (PSA)? Digital rectal exam A 50-year-old male comes to the nurse practitioner clinic for evaluation. He complains of chills, pelvic pain, and dysuria. He should be diagnosed with: acute bacterial prostatitis. A sexually active adolescent male reports that he has dysuria. How should this be managed? Urethral/urine cultures should be collected and screening done for STDs Neurological Disorders A patient complains of right leg numbness and tingling following a back injury. He has a diminished right patellar reflex and his symptoms are progressing to both legs. What test should be performed? Lumbar MRI The most common presenting sign of Parkinson’s disease is: tremor. falling. A 70-year-old male patient is diagnosed with vertigo. Which choice below indicates that the Persistent symptoms Nausea and vomiting vertigo is more likely to be of central etiology? A patient cannot stick his tongue out of his mouth and move it from side to side. What cranial nerve is responsible for movement of the tongue? Cranial Nerve XII Cranial Nerve X A 72-year-old patient with history of polymyalgia rheumatica complains of new onset, unilateral headache and visual changes. Her neurologic exam is otherwise normal. What is the most likely reason for her symptoms? Temporal arteritis Transient ischemic attack A patient with migraine headaches and hypertension should receive which medication class with caution? Triptans ACE inhibitors All of the following characteristics may be found in an older adult with dementia. Which one is common in a patient with Alzheimer’s disease, but uncommon in a patient with another type of dementia? Indifference Abrupt onset Sumatriptan (Imitrex) is a medication used to abort migraine headaches. It may also be used to treat: cluster headaches. depression. Which finding in a patient with migraine headache symptoms would compel the examiner Rapidly increasing Fully reversible to order an imaging study? intensity of headache speech disturbance A patient who had an embolic stroke has recovered and is performing all of her activities of daily living. Taking aspirin for stroke prevention is an example of: secondary prevention. quaternary prevention. Mrs. Johnson is an 89-year-old resident at a long- term care facility. Her state of health has declined rapidly over the past 2 months, and she can no longer make her own decisions. Her daughter requests a family conference with the nurse practitioner. Some important principles that need discussion at this time, if not previously documented, are: bereavement support for the family, quality of life for the resident, and living will. withdrawing therapy, hospice referral, and managing symptoms. An older adult patient has an audible carotid bruit. He has a history of hypertension, hyperlipidemia, and a myocardial infarction 5 years ago. He has no complaints today. The finding of a bruit indicates that the patient: is more likely to die from cardiovascular disease than cerebrovascular disease. is more likely to die from cardiovascular disease than cerebrovascular disease. Which finding below is typical in a patient who has Bell’s palsy? Unilateral numbness of the cheek Facial rash on the affected side A 2-month-old infant has an asymmetric Moro reflex. Which statement is true? The infant could have a birth injury. The infant could have a birth injury. What condition often causes a chief complaint of nocturnal paresthesias? Carpal tunnel syndrome Medial epicondylitis A patient diagnosed with cluster headaches: should eliminate triggers like nicotine and alcohol. may exhibit nuchal rigidity. Which characteristic is true of tension headaches, but not of cluster headaches? Tension headaches are always bilateral. Tension headaches cause photosensitivity. A neurologic disease that produces demyelination of the nerve cells in the brain and spinal cord is: multiple sclerosis. amyotrophic lateral sclerosis. The Snellen chart is used to assess: distant vision. peripheral vision. Which headache listed below is more likely to be triggered by food? Migraine Vascular A patient who is 82 years old is brought into the clinic. His wife states that he was working in his garden today and became disoriented and had slurred speech. She helped him back into the house, gave him cool fluids, and within 15 minutes his symptoms resolved. He appears in his usual state of health when he is examined. He states that although he was scared by the event, he feels fine now. How should the nurse Send him to the emergency department. Re-examine him tomorrow. practitioner proceed? A 70-year-old male who is diabetic presents with gait difficulty, cognitive disturbance, and urinary incontinence. What is part of the nurse practitioner’s differential diagnosis? Normal pressure hydrocephalus Multiple sclerosis Mrs. Jopson is unable to name a familiar object. How is this described? Anomia Incompetent What cranial nerve is responsible for hearing? VIII VIII Medications considered first line to treat attention deficit disorder and attention deficit hyperactivity disorder are Schedule: II. IV. A 52-year-old patient presents with an acute drooping upper right eyelid. She states that the right side of her face feels numb. Stroke has been ruled out. Based on the most likely etiology, how should she be managed? Steroids plus an antiviral agent Antiviral agent only A nurse practitioner is assessing the head of a 3- day-old infant. What would be a normal observation in a healthy 3-day-old infant who is crying? There are palpable pulsations over the anterior fontanel. The fontanel is depressed. A 60-year-old patient has anosmia. Which cranial I X nerve must be assessed? A patient complains of severe right-sided facial pain. She states that her symptoms have worsened over the past 48 hours. Which diagnosis below is NOT part of the differential diagnosis? Bell’s palsy Shingles When should medications be initiated in a patient who is diagnosed with Parkinson’s disease? When symptoms interfere with life’s activities After MRI and CT have ruled out stroke or tumor A patient is examined and found to have a positive Kernig's and Brudzinski's signs. What is the most likely diagnosis? Meningitis Pneumonitis A 62-year-old female patient presents to the clinic with very recent onset of intermittent but severe facial pain over the right cheek. She is diagnosed with trigeminal neuralgia. What assessment finding is typical of this? Pain may be triggered with light touch of the right cheek Pain is relieved with NSAIDs Mini mental status exam helps to identify patients who have symptoms of: cognitive impairment. stroke. Which choice below is a risk factor for sudden infant death syndrome (SIDS)? Low birth weight Female gender When is a child first able to stand on one foot? 3 years 4 years A patient reports a history of transient ischemic attack (TIA) 6 months ago. His daily medications are lisinopril, pravastatin, and metformin. After advising him to quit smoking, what intervention is most important in helping to prevent stroke in him? Taking low dose aspirin daily Assessing hemoglobin A1C every 3-6 months Which condition listed below does NOT impact an elder’s ability to eat? Hyperlipidemia Hyperlipidemia What is the earliest age that an average child would be able to copy a triangle, know his colors, and count on his fingers? 5 years 6 years A 68-year-old smoker with a history of well- controlled hypertension describes an event that occurred yesterday while mowing his lawn. He felt very dizzy and "passed out" for less than 1 minute. He awakened spontaneously. Today, he has no complaints and states that he feels fine. Initially, the NP should: perform a complete neurological and cardiac exam with auscultation of the carotid arteries. order a CT of the brain, blood clotting studies, and cardiac enzymes. A 75-year-old is diagnosed with essential tremor. What is the most commonly used medication to treat this? Long-acting propanolol Gabapentin Which of the following would NOT be part of the differential diagnosis for an 84-year-old patient with dementia symptoms? Normal aging process Normal aging process What recommendation should be made to an older adult who is diagnosed with mild dementia? The healthcare provider should recommend assessment of driving to determine risk of an accident. The patient may continue to drive as long as he feels comfortable. A patient who is 60 years old complains of low back pain for the last 5-6 weeks. She states that the severity is about 4/10 and that she gets no relief from sitting, standing, or lying. The NP should consider: systemic illness. disk disease. In a patient with end-of-life physical pain, constipation commonly occurs. What is the most common cause of this? Opioid use General slowing of body processes The “pill-rolling” tremor that is typical in patients with Parkinson’s disease is: an early manifestation of the disease. worse when the patient sleeps. Which reflex may be present at 9 months of age during sleep? Rooting Tonic neck Which symptom below is true of cluster headaches but not migraine headaches? The length of the headache duration is Sunlight is a common about 30-90 minutes. trigger. The cranial nerve responsible for vision is Cranial Nerve (CN): II. IV. An older adult patient is at increased risk of stroke and takes an aspirin daily. Aspirin use in this patient is an example of: primary prevention. primary or secondary prevention. The Mini-Cog is helpful in screening patients who have suspected: dementia. stroke. A person with 20/60 vision: has better vision than someone with 20/80 vision. has better vision than someone with 20/80 vision. The most common polyneuropathy in older adults is associated with: diabetes mellitus. Guillain-Barre syndrome. A 72-year-old patient with a history of coronary artery disease and hypertension reports an episode of slurred speech and right-sided facial droop that started yesterday while at home. It lasted for about an hour. She denies pain or headache. She presents to the clinic today and no longer has any of these symptoms. What is the most likely explanation for these symptoms? Transient ischemic attack Trigeminal neuralgia A 70-year-old patient exhibits a unilateral resting tremor. This likely indicates: Parkinson’s disease. benign essential tremor. In a 3-year-old with fever, which occurrence might precipitate a febrile seizure? A sudden decrease in body temperature after high fever Returning to daycare prior to being fever free for 24 hours A 74-year-old patient has a "pill-rolling" tremor. With what disease is this often associated? Parkinson’s disease Benign essential tremor Which class of medications is NOT used for migraine prophylaxis? Triptans Tricyclic antidepressants A typical description of sciatica is: burning and sharp. precipitated by coughing. A 26-year-old HIV-positive patient presents with photophobia and temperature of 103.2° F. He complains of a headache. On exam, he is unable to demonstrate full extension of the knee when his hip is flexed. Which choice below is the most likely diagnosis? Meningitis Septic bursitis A patient has developed loss of hearing over the past several weeks. His otoscopic exam is normal. What cranial nerve should be assessed? Cranial Nerve VIII Cranial Nerve X Restless legs syndrome is part of the differential Serum ferritin Urinalysis diagnosis for Mr. Wheaton. What should be part of the laboratory workup? A family member of a newly diagnosed Alzheimer’s disease patient asks how long the patient should take donepezil (Aricept), an acetylcholinesterase inhibitor, before learning whether it is beneficial or not. You reply: 6 - 12 months. 6 - 12 months. A young male patient with a herniated disk reports bilateral sciatica and leg weakness. If he calls the NP with complaints of urinary incontinence, what should be suspected? Cauda equina syndrome Cauda equina syndrome A child who can stack a maximum of five blocks is probably: 2 years of age. 3 years of age. An 80-year-old patient comes into the clinic with an ataxic gait, complaints of new-onset headache and slurred speech that began about 2 hours ago. What is the likely etiology of this event? Stroke or TIA Bell’s palsy What is the earliest age that an average child would appropriately receive construction paper and a pair of scissors with rounded points? 4 years 6 years Mr. Williams has moderate cognitive deficits attributed to Alzheimer’s disease and has been started on a cholinesterase inhibitor. The slow progression of his cognitive deficits. slow progression of his cognitive deficits. purpose of this drug is to: Which of the following are diagnostic criteria for migraine headache without aura? Pain lasts 4-72 hours Photophobia must be present Which factor listed below does NOT contribute to the risk of falls in older adults? Decreased hearing Anticholinergic medications A patient is diagnosed with carpal tunnel syndrome. Which finger is not affected by carpal tunnel syndrome? Fifth finger Fifth finger Cranial nerve II could be evaluated in a young infant by assessing: squinting response to bright light. rapid eye movement. Asking the patient to visually follow a finger through the cardinal fields of gaze assesses which cranial nerve? III V An older adult patient with organic brain syndrome is at increased risk of abuse because she: has declining cognitive function. has declining cognitive function. Which cranial nerve is assessed by administering the Snellen test? II VI The "get up and go" test in an older adult patient is used to evaluate: risk for falls. driving safety. Orthopedics Disorders A college age basketball player landed awkwardly on his foot and ankle after jumping during a game yesterday. He states that he sprained his ankle. He complains of ankle pain and foot pain but is able to limp into the exam room. How should he be managed? Nonweightbearing until fracture is ruled out Women are commonly screened for osteoporosis at age 65 years. How should screening for osteoporosis be managed in a male this age? They should be screened starting at age 65 if they have risk factors. Ankle inversion is a common complaint from a patient with a: lateral ankle sprain. A male patient takes HCTZ daily for hypertension. He developed severe pain in his great toe yesterday. He was diagnosed with gout today and started on a medication. Which medication listed below would be contraindicated at this time? Allopurinol A 60-year-old adult with an antalgic gait and complaints of hip pain is examined. He has trochanteric tenderness. What is the most common cause of this? Trochanteric bursitis What is the most prevalent skeletal problem in the United States? Osteoarthritis A patient with sciatica is most likely to describe relief of symptoms with: side lying or standing. The drop arm test is used to assess patients with suspected: rotator cuff injury. A 16-year-old male plays trumpet in the school marching band. He has had marching practice every day for the last week. Today he complains of pain in his left midfoot. The foot is neither swollen nor red. What is the most likely diagnosis in the differential? Stress fracture A man fell off a 3-foot stepladder while working at home. He presents to your office with complaints of foot pain. He has point tenderness over the medial malleolus and swelling, but he is able to ambulate. How should this be managed? X-ray of the foot and ankle An older adult had a total knee replacement (TKR) 10 months ago. He is doing well. He states that he is having a dental procedure performed in 1 week. What information should he be given regarding the knee replacement? He should not routinely receive an antibiotic prior to dental procedures. Which of the following tests, if positive, is part of the criteria for systemic lupus erythematosus (SLE)? Antinuclear antibody (ANA) A 75-year-old patient has osteoarthritis and pain. Which of the following medications increases the risk of a GI-related ulceration? Celecoxib Which of the following is NOT true regarding cervical whiplash injury? Is identifiable on MRI or CT, but not X-ray. Which factor listed below is NOT considered a risk factor for development of osteoporosis? Late menopause An adolescent athlete has sprained his ankle. What instruction should be given to him regarding activity? He should be able to walk pain-free before he starts to run. A 55-year-old male patient describes severe pain at the base of his left first toe. He is limping and says he can’t remember hurting his toe. Which symptom below suggests Fever something other than gout? When should functional rehabilitation occur once a patient has had an ankle or knee sprain? The day of the injury A long distance runner is diagnosed with a tibial stress fracture. Which statement is true about the injury? Rest and an alternative activity are recommended. A patient is at increased risk of osteopenia if she uses which form of birth control? Injectable progestin What does a positive anterior drawer test demonstrate in a patient who has an injured knee? Instability of the anterior cruciate ligament A 75-year-old has pain from osteoarthritis in her right knee. What intervention is considered first line to treat her pain? Exercise Management of a sprained ankle includes: rest, ice, compression, elevation. What should the nurse practitioner assess in an 80-year-old patient who reports a fall but does not have serious physical injury? Medications taken a few hours before the fall Which symptom can be used to rule out a fracture? No symptom can rule out a fracture A 75-year-old female who knits daily has a positive Finkelstein test. What is her likely diagnosis? De Quervain's tenosynovitis A patient who frequently has episodes of gout should avoid which groups of food? Roast beef with gravy, rice Which joints are least commonly involved in osteoarthritis? Wrists A characteristic of rheumatoid arthritis not typical in osteoarthritis is: symmetrical joint involvement. A positive Tinel’s test can be used to assess carpal tunnel syndrome. What other test can be used to assess for this? Phalen’s test A patient complains of pain in his right heel. Initially, it began with his first steps out of bed in the morning, but over the past few weeks he has heel pain throughout the day. What factor has contributed most to his worsening heel pain? Pes planus A 66-year-old African American female has Her race multiple risk factors for osteoporosis. Which choice listed below is NOT a risk factor for osteoporosis? Bone mineral density screening in women older than 65 years is an example of: secondary prevention. A 60-year-old female presents with history of low back pain of recent origin. Her gait is antalgic and she reports loss of bladder function since the onset of back pain this morning. What should be done? Refer to the emergency department If plantar fasciitis is suspected in a patient, how is this diagnosed? Physical exam Which findings are most commonly associated with rheumatoid arthritis? Morning stiffness, positive rheumatoid antigen and antinuclear antibody What is the value of vitamin D supplementation in the diet of older adults? It decreases the risk of falls. A patient cut himself on a fence post while working outside. He has not had a tetanus shot in more than 10 years. How long can he wait before getting the immunization and still 72 hours prevent tetanus? A 79-year-old frail adult reports that she had a fall last week. She had no broken bones but is very sore. In evaluating this adult, what question is most important to ask? Have you had other falls this year? A 45-year-old male who is in good health presents with complaints of pain in his left heel. He states that the first few steps out of bed in the morning are extremely painful. He has no history of trauma. What is the likely etiology of his pain? Plantar fasciitis A 60-year-old has been on NSAIDs and hydrocodone for the past 48 hours for shoulder pain. He has complaints of blood on toilet tissue when wiping after a bowel movement this morning. What should be suspected? He could have hemorrhoids. Which statement below is true regarding NSAIDs for low back pain? They are associated with more side effects than acetaminophen. Which patient below should be screened for osteoporosis? 60-year-old male with rheumatoid arthritis The Ottawa ankle rules help the examiner determine when: X-rays are needed with a suspected ankle sprain. An 80-year-old patient is very active but presents today with posterior hip pain for the past week. Which of the following is least likely part of the differential diagnosis? Osteoarthritis Which of the following is true about metatarsus adductus? Mild flexible metatarsus usually spontaneously corrects. A patient reports that her knee “locks” sometimes and feels like it will “give out.” She denies injury. She has no complaint about her other knee. What is her likely problem? Meniscal tear A patient has suspected plantar fasciitis. The plantar fascia is best examined: with the great toe dorsiflexed. A patient presents with right shoulder pain (7/10 on the pain scale) after an acute shoulder injury yesterday. He fell against a brick wall while working at his home. He reports pain that radiates into his upper arm. How should this be managed? Order an X-ray of the right shoulder What medication is recommended by the American College of Rheumatology as a first line agent for a patient who has been unsuccessful with nonpharmacologic interventions for osteoarthritis pain? Acetaminophen Which diagnosis is the least likely cause of extrinsic shoulder pain? Gout A 62-year-old female presents with atraumatic right knee pain. On exam, she has a mildly swollen right knee that is not warm or tender to touch. She has a negative McMurray test. An x-ray was obtained. How should the nurse practitioner interpret the results of this X-ray? Osteoarthritis An adolescent athlete has injured his ankle playing basketball. He has right ankle pain, ecchymosis, and significant edema, and he is unable to bear weight at the time of the clinical exam. Which diagnosis is least likely? Grade I sprain A 65-year-old male is diagnosed with an initial episode of gout. It is likely that he: will have severe inflammation in a single joint. A young athlete is found to have a depression of the longitudinal arch of both feet. He complains of bilateral heel pain. The rest of his He needs some heel support in his shoes. foot is normal and he has continued with his activities. What could be recommended for his heel pain? A nurse practitioner has successfully reduced a nursemaid’s elbow. How can the NP know that it was successful? Child moves the affected arm at the elbow A 70-year-old patient has had intermittent back pain secondary to a bulging disk for more than 3 years. In the last year, it is constant (pain scale is 2-3/10) and at times is sharp. She is not a surgical candidate. What class of medication would be a good choice for improvement of chronic pain in this patient? Gabapentin A 14-year-old male client reports dull anterior knee pain, exacerbated by kneeling. What is the likely etiology? Osgood-Schlatter disease A 40-year-old complains of back pain after heavy lifting. This began 2 weeks ago. He has had little improvement in his pain. Which statement is true regarding plain X-rays in this patient? X-rays usually detect findings unrelated to symptoms. A 6-year-old complains that his legs hurt. His mother states that he has complained for the past 2 weeks, and she thought it was from Complete blood count “playing outside too much”. When asked to identify the painful areas, the child points to the midshaft of the femurs. He grimaces slightly when asked to walk. What is the most important initial intervention? A 2-year-old was brought into the clinic by her mother who reports that she pulled her arms upward to pick her up and now the child won’t use her right arm. A nursemaid’s elbow is suspected. Which statement below is correct? Her arm is slightly flexed and held close to her body. A 50-year-old patient reports acute pain in his lower back that started 2 weeks ago after working in his yard. The pain radiates into his right leg intermittently. He has been managing his pain with ibuprofen. There are no red flags in his history or on exam. When should consideration be given to imaging studies? At 4 weeks Which statement below best characterizes scoliosis in an adolescent? There can be unequal rib prominences or shoulder heights. A 65-year-old female complains of left medial knee pain. She has been told that she has arthritis in this knee. Where would the pain be located? Medially, along the joint line An adolescent complains of knee pain. He is diagnosed with Osgood-Schlatter disease. What assessment finding is typical? Pain worsens with quadriceps contraction. A 60-year-old patient who is otherwise healthy presents with acute onset of right knee pain. She denies injury but reports that she walked up a lot of steps yesterday. She is diagnosed with prepatellar bursitis. What is a common finding? Swelling and pain to touch of the anterior knee A positive Trendelenburg’s test could be used to identify a child with: slipped capital femoral epiphysis. A high purine diet can contribute to gouty arthritis. Which food listed below contributes most to a high purine diet? Beef When is Osgood-Schlatter disease most likely to produce symptoms? During a growth spurt A male patient who injured his back lifting a heavy object reports that he has low back pain. He is diagnosed with a lumbar strain. He is afraid to continue activities of daily living, especially walking, because he has pain with these activities. What statement below is true? Gradually increase activities of daily living and walking as tolerated. A 49-year-old patient has osteoarthritis in the lumbar spine and hip. His hip X-ray demonstrates bone on bone. What can be done to resolve his complaints of pain in his hip? Acetaminophen and a referral to orthopedics An older adult with a complaint of shoulder pain has a positive “drop arm” test. What is his likely diagnosis? Torn rotator cuff A 72-year-old patient complains of knee pain when she climbs stairs or walks long distances. Crepitus is palpable in the affected knee. What is the likely cause? Arthritis of the knee A 75-year-old female who is otherwise healthy has mild osteoarthritis in her right knee. She complains of pain not relieved by acetaminophen 2000 mg daily. What should be done? Continue acetaminophen and order physical therapy Clubfoot: involves the foot and lower extremity. A 6-year-old complains that his legs hurt. His mother states that he has complained for the past 2 weeks, and she thought it was from “playing outside too much”. When asked to identify the painful areas, the child points to Acute lymphocytic leukemia (ALL) the midshaft of the femurs. He grimaces slightly when asked to walk. What should be part of the differential diagnosis? Which beverage below does not increase the risk of gout in a male who is prone to this condition? Wine An 8-year-old has a painful limp. He reports that his knee hurts medially. On exam he has pain with internal rotation of the hip. How should the NP manage this situation? The NP should order a hip X-ray, CBC and ESR. A 16-year-old complains that his knees hurt. His mother states that he has complained of knee pain for the past 2 weeks. He has a prominent tibial tubercle. What should be part of the differential diagnosis? Osgood–Schlatter disease A 12-year-old male with hip pain presents to the NP clinic. Hip pain has occurred with activity for the past 4-6 weeks, but his pain is worse and now involves the knee. There is no history of trauma. How should the workup be initiated? Perform Trendelenburg’s test in the office A 70-year-old African American male complains of pain in his back and trunk. Cardiovascular disease is ruled out. Laboratory multiple myeloma. studies were obtained. A likely diagnosis is: The age at which a child can first walk backwards is: 18 months. The average age of pubertal growth spurt in North American boys is: 12 - 14 years. Pediatric Disorders A child's resting heart rate is expected to be between 60 and 100 beats per minute once he reaches:10 years of age. 10 years of age. Which of the following is true about metatarsus adductus? Mild flexible metatarsus usually spontaneously corrects. A 1-week-old infant has a bilateral mucopurulent eye discharge. What explains the etiology of the discharge? The mother probably has an STD. Children with an inguinal hernia: have a history of an intermittent bulge in the groin. A 4-month-old is suspected of having Galeazzi and Klisic developmental dysplasia of the hip (DDH). What test would best assess for this? A 15-year-old female has been sexually active since she was 12 years old. She presents today with concerns of pregnancy. How should this be managed? Pregnancy test should be performed today. An 18-month-old child is diagnosed with bronchiolitis. His respiratory rate is 28 breaths per minute. Which choice below is most appropriate for patient management? Antipyretics A 7-year-old entered the clinic 1 month ago. There was no evidence that he had any immunizations. He was given the vaccinations listed on his vaccination record at the time of his visit. If he returns today, which immunizations can he receive? Hepatitis B, DTaP, IPV, MMR A 3-year-old has been recently treated for an upper respiratory infection (URI) but drainage from the right nostril persists. What should the NP suspect? Presence of a foreign body The patient presents with complaints of morning eyelash crusting and itchy red eyes. It began on the left and now has become bilateral. Based on the most likely diagnosis, what should the nurse This usually begins as a viral infection. practitioner tell the caregivers about this condition? An adolescent takes isotretinoin for nodulocystic acne. She is on oral contraceptives. Both were prescribed by the dermatologist. The adolescent presents to your clinic with a sinus infection. Her temperature is 99.5° F and her blood pressure is 160/100 mmHg. How should this be managed? Call the dermatologist to report the elevated BP A definitive diagnosis of sickle cell anemia can be made: by a hemoglobin electrophoresis. A 4-month-old infant has thrush. The mother is breastfeeding. She reports that her nipples have become red, irritated, and sensitive. What should the nurse practitioner advise the mother of this baby to treat thrush? Treat the infant with an oral antifungal suspension and the mother’s nipples with a topical antifungal agent Which reflex may be present at 9 months of age during sleep? Rooting A pregnant mother in her first trimester has a 5- year-old who has Fifth Disease. What implication does this have for the mother? There is a risk of fetal death if she becomes infected. The pneumococcal immunization in infants has: shifted the pathogenesis to fewer cases of S. pneumoniae. A healthy appearing 3-year-old female presents with nonblanchable redness over both knees and elbows. During the exam, she is found to have normal growth and development, and she interacts appropriately with the nurse practitioner. She had an upper respiratory infection about 4 weeks ago that cleared without incident. A CBC and urinalysis were obtained. The most likely diagnosis is: idiopathic thrombocytopenia purpura (ITP). The test of choice to confirm and assess developmental dysplasia of the hip (DHH) in a 3- month-old is: ultrasound of the hip. What is the usual age for vision screening in young children? 3 years A 6-year-old is brought to your clinic because of behavior problems at school. DSM V criteria are used to diagnose attention deficit disorder (ADD). Which finding is likely present? Inattention A 7-year-old has a complaint of ear pain. If he has otitis externa, which complaint is most likely? He has tragal pain. A 15-year-old male has worked this summer as a lifeguard at a local swimming pool. He complains well marginated half- moon macules on the of itching in the groin area. He is diagnosed with tinea cruris. The nurse practitioner is likely to identify: inner thigh. The initial step in the management of encopresis is: client and family education. What would be age appropriate anticipatory guidance for the parent of a 9-month-old infant? Discuss weaning from a bottle. Babies up to one year of age are called: infants. A 6-week-old is suspected of having developmental dysplasia of the hip (DDH). What test would best assess for this? Barlow and Ortolani Which of the following is an appropriate initial intervention for gastroesophageal reflux disease (GERD) in an 8-week-old? Small, frequent thickened feedings Most hypertension in preadolescents and children is: secondary hypertension. A sexually active adolescent male reports that he has dysuria. How should this be managed? Urethral/urine cultures should be collected and screening done for STDs An NP examines a screaming 2-year-old. A common finding is: pink tympanic membranes. According to the Tanner stages of development (Sexual Maturity Rating), breast development in Stage 3 in females includes the: enlargement and elevation of breast and areola with no separation of their contours. A mother presents with her 1-month-old infant. She reports that he cries inconsolably every evening after his first evening feeding. She asks for help. What should be done? Provide education, parental reassurance, and encouragement. An 8-year-old has a painful limp. He reports that his knee hurts medially. On exam he has pain with internal rotation of the hip. How should the NP manage this situation? The NP should order a hip X-ray, CBC and ESR. The nurse practitioner is examining a 3-month- old infant who has normal development. She has identified an alopecic area at the occiput. What should be done? Encourage the caregiver to change the infant’s head position A young female has breast buds bilaterally. This represents Tanner Stage: II A 3-day-old infant weighed 8 pounds at birth. Today he weighs 7.5 pounds. How should this be managed? Continue feeding every 2- 4 hours What early finding may lead the provider to suspect renal artery stenosis in a 3-year-old male? Increased blood pressure A 2-year-old with sickle cell anemia (SCA) should receive which immunizations? All routine childhood immunizations at the usual time The nurse practitioner identifies satellite lesions in a 6-month-old infant. These are: indicative of candidal infection. A 6-week-old infant is found to have positive Barlow and Ortolani maneuvers. What test should the NP order to assess and confirm developmental dysplasia of the hip (DDH)? Ultrasound of the hip A 3-year-old has fluid in the middle ear that does not appear infected. The eardrum appears normal. This is referred to as: middle ear effusion. A 6-day-old has bilateral mucopurulent eye discharge. Which historical finding explains the etiology of the discharge? Mother has chlamydia A child received a burn on his chest from a cup of hot coffee. On examination, the injured area appeared moist, red to ivory white in color, and features blisters. It is painful to touch. This burn would be classified as a: superficial partial thickness burn. Hirschsprung's disease is characterized by: failure to pass meconium in the first 48 hours of life. The average age of pubertal growth spurt in North American boys is: 12 - 14 years. The nurse practitioner sees a child who reports fatigue and presents with purpura on his lower extremities. His temperature is normal. The differential includes: acute leukemia. Medications considered first line to treat attention deficit disorder and attention deficit hyperactivity disorder are Schedule: II. The Sexual Maturity Rating (Tanner Staging) in boys includes 5 stages. Characteristics indicative of Stage 1 would include: the absence of pubic hair; same size and proportion penis, testes, and scrotum as in childhood. The most common cause of pneumonia in an otherwise healthy 3-year-old child is: a viral infection. The term caput succedaneum refers to: scalp edema. An adolescent complains of knee pain. He is diagnosed with Osgood-Schlatter disease. What assessment finding is typical? Pain worsens with quadriceps contraction. Which of the following increases the risk of cryptorchidism? Premature birth At what age should oral health risk assessment begin? 6 months Which patient below is most likely to experience stranger anxiety during a physical exam? A 12-month-old female What is the earliest age that an average child would appropriately receive construction paper and a pair of scissors with rounded points? 4 years A healthy 7-year-old child is diagnosed with atypical pneumonia. He is febrile but not in distress. What is the preferred treatment for him? Azithromycin A 2-year-old has a sudden onset of high fever while at daycare. The daycare attendant describes a seizure in the child. The child is brought to the clinic; neurologically he appears Treat the otitis media and give education about fever management normal. His body temperature is 99.9° F after receiving ibuprofen. He is diagnosed with otitis media. How should the nurse practitioner manage this? According to the Tanner stages of development (Sexual Maturity Rating), breast development in Stage 4 in females includes the: projection of areola and nipple to form a secondary mound above the level of the breast. A 12-month-old was screened for iron deficiency anemia and found to be anemic. The nurse practitioner ordered oral iron. In 1 month, the child’s hemoglobin was reassessed. It increased greatly. Which answer might account for this? The bone marrow is responding appropriately. A male child has swelling in the upper left thigh below the inguinal ligament. He complains of cramping and intermittent lower abdominal pain. What should be included in the differential diagnosis? Femoral hernia What advice should be given to a parent who has a child with Fifth Disease? A parent may experience joint aches and pains. A 5-year-old female’s playmate has been diagnosed with pinworms. The mother brings her child in for an exam. The 5 year-old denies rectal itching. How should the NP proceed? Perform the “scotch tape” test and look at the collection under the microscope. The age at which a child can first walk backwards is: 18 months. A child who can stack a maximum of five blocks is probably: 2 years of age. Which choice below is a risk factor for sudden infant death syndrome (SIDS)? Low birth weight Which statement about attention deficit disorder (ADD) is correct? DSM V is used to diagnose a child with ADD. A 9-year-old has been diagnosed with chickenpox. A drug that should be avoided in him is: aspirin. According to the Tanner stages of development (Sexual Maturity Rating), breast development in Stage 5 in females includes the: projection of the nipple only. A congenital heart abnormality often discovered during the newborn period is coarctation of the aorta. How is this assessed? By comparing upper and lower extremity blood pressures A 6-year-old complains that his legs hurt. His mother states that he has complained for the past 2 weeks, and she thought it was from “playing outside too much”. When asked to Acute lymphocytic leukemia (ALL) identify the painful areas, the child points to the midshaft of the femurs. He grimaces slightly when asked to walk. What should be part of the differential diagnosis? A 6-month-old child comes into the clinic for immunizations. Which item below allows a delay in his getting immunizations today? Child has otitis media with temperature of 103° F A 6-month-old male has a palpable cystic mass in his scrotum. His mother states that sometimes the size of his scrotum varies. What should the nurse practitioner do next? Referral to urology if this has not resolved in 6 more months Pharmacologic treatment for children who have hypertension should be initiated for: diabetics with hypertension. A 7-year-old enters the nurse practitioner clinic. There is no evidence that he has received any immunizations. What should be administered today? Hepatitis B, Tdap, IPV, varicella, MMR What Tanner stage corresponds to an average 8- year-old male? Stage 1 When is Osgood-Schlatter disease most likely to produce symptoms? During a growth spurt A 4-year-old was diagnosed and treated for acute otitis media in the left ear 4 weeks ago. She is here today for a well-child visit. There is an effusion in the left ear. She denies complaints. How should this be managed? This should be monitored. Hand-foot-and-mouth disease and herpangina: are viral infections caused by Coxsackie viruses. At what age would it be unusual to see thrush? At birth A 6-year-old being treated for community- acquired pneumonia (CAP) has been taking azithromycin in therapeutic doses for 72 hours. His temperature has gone from 102° F to 101° F. What should be done? Change antibiotics to a penicillin A full-term newborn is diagnosed with hyperbilirubinemia. When would his bilirubin level be expected to peak? 3-4 days A pregnant patient is concerned because her 12 month-old needs an MMR immunization. What should the NP advise this patient? MMR immunization presents no risk to the child’s mother. Immunize now. A 3-year-old has been diagnosed with acute otitis media. She is penicillin allergic (Type 1 hypersensitivity reaction). How should she be Clarithromycin managed? A 14 year-old male patient has an acute, painless groin swelling. Which tool would yield the most information to identify the etiology of the swelling? Ultrasound of the scrotum A positive Trendelenburg’s test could be used to identify a child with: slipped capital femoral epiphysis. A nurse practitioner has successfully reduced a nursemaid’s elbow. How can the NP know that it was successful? Child moves the affected arm at the elbow A 7-year-old male presents with encopresis. The NP might expect: constipation. Which statement below best characterizes scoliosis in an adolescent? There can be unequal rib prominences or shoulder heights. What is the earliest age that an average child would be able to copy a triangle, know his colors, and count on his fingers? 5 years Cranial nerve II could be evaluated in a young infant by assessing: squinting response to bright light. Head circumference should be measured until a child has attained: 36 months of age. A 6-year-old complains that his legs hurt. His mother states that he has complained for the past 2 weeks, and she thought it was from “playing outside too much”. When asked to identify the painful areas, the child points to the midshaft of the femurs. He grimaces slightly when asked to walk. What is the most important initial intervention? Complete blood count The chest circumference of a 12 month-old is: equal to head circumference. In a 3-year-old with fever, which occurrence might precipitate a febrile seizure? A sudden decrease in body temperature after high fever The most common cause of acute pharyngitis in children is: respiratory viruses. A 12-year-old complains of itching in his right ear and pain when the pinna is pulled or the tragus is pushed. Examination revealed slight redness in the ear canal with a clear odorless fluid. This could be suggestive of: otitis externa. A 3-year-old female had a fever of 102° F for the last 3 days. Today she woke up from a nap and is afebrile. She has a maculopapular rash. Which statement is true? The rash will blanch. Which statement below is true of infants with developmental dysplasia of the hip? A palpable clunk is considered diagnostic. When is a child first able to stand on one foot? 3 years A young child has developed a circumferential lesion on her inner forearm. It is slightly raised, red and is pruritic. It is about 2.5 cm in diameter. This is probably related to: the child’s new cat. A 6-week-old male infant is brought to the nurse practitioner because of vomiting. The mother describes vomiting after feeding and feeling a “knot” in his abdomen especially after he vomits. The child appears adequately nourished. What is the likely etiology? Pyloric stenosis An otherwise healthy 6-year-old male has been diagnosed with otitis media. His mother reports that he has not had an ear infection since he was 3 years old. How long should he be treated with an 5-7 days antibiotic? An adolescent male has had a sudden onset of severe scrotal pain following a kick in the groin earlier in the day during a soccer game. How should this be managed? He should be referred to the emergency department. Many factors can contribute to the risk of congenital heart disease. Which maternal disease carries a higher risk of transposition of the great vessels (TGA), ventricular septal defect (VSD), and hypertrophic cardiomyopathy? Diabetes Which suggestion below is the standard for treating iron deficiency anemia in infants and children? Iron supplementation in divided doses between meals with orange juice An infant is diagnosed with diaper dermatitis. Satellite lesions are visible. This should be treated with a: topical antifungal agent. Epstein-Barr virus is responsible for: mononucleosis. An adolescent has acne. The nurse practitioner prescribed a benzoyl peroxide product for him. What important teaching point should be given to this adolescent Photosensitivity of the skin can occur regarding the benzoyl peroxide? A child with a sandpaper-textured rash probably has: Strept infection. A 1-year-old patient’s mother reports allergy to gelatin. The mother describes the reaction as “lips swelling and breathing difficulties that necessitated a trip to the emergency department.” Which immunizations should be avoided? Varicella and MMR Risk assessment for dyslipidemia should begin at: 2 years. A patient has suspected scarlet fever. He likely has a sandpaper rash and: a positive rapid Strept test. At what age should initial blood pressure screening take place? 3 years A 4-year-old child with otitis media with effusion: probably has just had acute otitis media. MMR immunization is safe in children: who are allergic to eggs. Which of the following will decrease the Breastfeeding risk of acute otitis media in a 6-month-old? Which infant feeding behavior is least likely related to congenital heart disease (CHD)? Infants that burp frequently when feeding A mother reports that her child is not allergic to chickens but is allergic to ducks and duck feathers. The child is 4 years old today. Which immunizations should he receive? None are contraindicated An infant is brought to the nurse practitioner because his gaze is asymmetrical. Which finding indicates a need for referral to ophthalmology? He has persistent strabismus. How often should blood pressure be measured in a child who is 3 years old? It should be measured annually. When does a child’s vision approximate 20/20? 5-6 years A child who is 15 months old is referred to as a(n): toddler. A 9-year-old female has presented to your clinic because of a rash on the left, upper Examine all other areas of the trunk, then ask area of her anterior trunk. She is embarrassed and very reticent to lift her blouse because her nipple will be exposed. How should the NP proceed? the child to lift her blouse A 3-day-old full-term infant has a bilirubin level of 16 mg/dL. How should this be managed? Order phototherapy for the infant A tympanic membrane (TM) is erythematous. Which factor listed below is NOT the cause of an erythematous TM? Coughing A 12-month-old is here today to receive the varicella immunization. A patient’s mother reports that her 12-month-old child was exposed to chickenpox about a week ago. The NP should recommend that he: be given the vaccine. A 2-month-old is diagnosed with thrush. An exam of this patient’s saliva demonstrates all except: spores. Which reflexes might a 1-month-old infant be expected to exhibit? Moro, stepping, rooting A young child has an audible murmur. The nurse practitioner describes it as a grade 4 The child should be murmur. How should this be managed? referred to cardiology. A 2-month-old infant has an asymmetric Moro reflex. Which statement is true? The infant could have a birth injury. When can a child with chickenpox return to daycare? After all lesions have crusted A 2-year-old was brought into the clinic by her mother who reports that she pulled her arms upward to pick her up and now the child won’t use her right arm. A nursemaid’s elbow is suspected. Which statement below is correct? Her arm is slightly flexed and held close to her body. The term that describes the urethral opening on the ventral surface of the penis is: hypospadias. A 10-year-old female presents with a 3- month history of abdominal pain. She has been diagnosed with recurrent abdominal pain. During the interview the nurse practitioner is likely to elicit a finding of: school absenteeism. A nurse practitioner is assessing the head of a 3-day-old infant. What would be a normal observation in a healthy 3-day-old infant who is crying? There are palpable pulsations over the anterior fontanel. A child has 8-10 medium brown café au lait spots > 1 cm in diameter. The differential diagnosis should include: neurofibromatosis. A 12-year-old male with hip pain presents to the NP clinic. Hip pain has occurred with activity for the past 4-6 weeks, but his pain is worse and now involves the knee. There is no history of trauma. How should the workup be initiated? Perform Trendelenburg’s test in the office A 4-year-old female is brought in to the clinic by her mother, who reports that she is constantly scratching “her private part.” The patient states that it itches. On exam, the vagina is red and irritated. How should the NP proceed? Collect a vaginal swab of the external vagina for microscopic evaluation A 4-year-old is being examined today in the NP clinic. He appears shy and does not make eye contact with the examiner. The mother does not make eye contact with the examiner either. The patient lacks animation and does not smile. What likely Neglect is possible. possibility must be considered? What is the usual recommendation about administering MMR and varicella immunizations? They should be given on the same day or at least one month apart. A 6-year-old has been diagnosed with Lyme disease. Which drug should be used to treat him? Amoxicillin The first sign that a male child is experiencing sexual maturation is: increase in testicular size. A 6-month-old infant has a disconjugate gaze. The nurse practitioner observes that the 6-month-old tilts his head when looking at objects in the room. Which statement is true? The infant will have an abnormal cover/uncover test. Clubfoot: involves the foot and lower extremity. A mother of a 4-week-old infant visits your office. She states that her baby is vomiting after feeding and then cries as if he is hungry again. What should the nurse practitioner assess? His abdomen for an olive shaped mass A 6-year-old patient with sore throat has coryza, hoarseness, and diarrhea. What is the likely etiology? Viral etiology A 6-year-old child who has moderate persistent asthma is diagnosed with pneumonia after chest X-ray and laboratory studies. He developed a sudden onset of fever with chills. He is in no distress. What is the preferred treatment for him? Amoxicillin A 4-year-old has been diagnosed with measles. The nurse practitioner identifies Koplik’s spots. These are: found on the inside of the cheek and are granular. A sexually active adolescent male has a warty growth on the shaft of his penis. It is painless. This is likely: human papilloma virus. Pregnancy Disorders Ultrasounds are commonly performed during the first trimester of pregnancy because they help estimate gestational age and: identify fetal malformations. Few pregnant patients actually deliver on their due dates. Why is a due date To determine timing of maternal/fetal established? screenings A 30-year-old female presents with lower abdominal pain. The nurse practitioner immediately considers an ectopic pregnancy as the cause. Which factor listed below does NOT increase her risk of an ectopic pregnancy? Age Which immunization(s) may be safely given during the third trimester of pregnancy? Tdap A pregnant patient in her second trimester will probably have a decrease in her: blood pressure. In order to establish pregnancy, a pregnancy test of the urine or blood is routinely performed. How early can this be done with reliable results? 6-8 days after conception A pregnant patient asks if engaging in sexual activity will place her fetus at increased risk. The nurse practitioner responds: this may increase the risk of preterm labor. A pregnant patient is in her first trimester in October. How should a flu immunization be handled for her? Give the influenza immunization without regard to trimester. A 17-year-old female is found to be pregnant. What is the LEAST likely risk to her fetus? Down Syndrome A patient in her first trimester of pregnancy is found to have gonorrhea. Which statement below is true? She should be treated now for gonorrhea and chlamydia. In order to establish pregnancy, a pregnancy test of the urine or blood is routinely performed. This test assesses for: presence of beta hCG. Which of the following immunizations may be safely administered during the first trimester of pregnancy? Inactive influenza A patient who is found to be pregnant has asymptomatic bacteriuria. How should this be managed? Prescribe nitrofurantoin A 26-year-old pregnant female has complains of dysuria. She has the following urinalysis report. Which statement below is true? The patient has a urinary tract infection and E. coli is the most likely pathogen. The classic symptoms of an ectopic pregnancy are: amenorrhea, vaginal bleeding, abdominal pain. A pregnant patient in her second trimester enters your clinic in October to inquire about the influenza vaccination. Which immunization below may be safely administered? Inactivated injection The need for thyroid replacement during pregnancy: increases. In a viable pregnancy: fetal heart tones are audible at about 9-12 weeks. Hyperemesis gravidarum is: persistent, intractable vomiting during pregnancy. The NP suspects that a pregnant patient may have been physically abused by a domestic partner. The NP knows that: abuse can worsen during pregnancy. The classic presentation of placenta previa is: painless vaginal bleeding after the 20th week. A patient in her first trimester of pregnancy is found to be infected with chlamydia and gonorrhea. Which statement below is true? She should be treated now and rescreened later in pregnancy. A pregnant patient complains of lower extremity edema and asks for a “fluid pill”. The NP explains that: this is best treated with rest and elevation of the legs. A female who is being counseled preconceptually is found to have a negative rubella titer. If she is immunized today, for how long should she avoid pregnancy? 1 month A pregnant teenager asks if sexual activity is safe during pregnancy. The nurse practitioner responds: this can expose you to STDs. The rubella vaccine is contraindicated in pregnant women because: it can cause rubella in the infant. A patient with an ectopic pregnancy: will have a positive pregnancy test. A pregnant patient is found to have positive leukocytes and positive nitrites in her urine. She is asymptomatic. What medication should be given? Nitrofurantoin The Centers for Disease Control and Prevention (CDC) recommends Tdap for every pregnant woman. When should this vaccine be administered? 3rd trimester A 24-year-old pregnant patient has a TSH performed. The most likely reason for this is because: she has hypothyroidism. A 19-year-old pregnant female has the following urinalysis report. Which statement below is true? This patient has a contaminated specimen A patient was diagnosed today with pregnancy. Her last pregnancy was 3 years ago. At that time she had a protective rubella titer. What should be done about evaluating a rubella titer today? She does not need one because it was protective 3 years ago. Naegele's rule is calculated by adding 7 days to the last menstrual period and: subtracting 3 months. A pregnant patient in her first trimester is found to have chlamydia. How should this be managed? Treat with azithromycin. Which factor listed below increases the risk of ectopic pregnancy? Prior history of ectopic pregnancy Routine screening for gestational diabetes: takes place at about 24- 28 weeks. You have been asked to manage thyroid She should continue it disease in a pregnant patient. A pregnant patient took levothyroxine prior to becoming pregnant. What should be done about the levothyroxine now that she is pregnant? and have monthly TSH levels. A 24-year-old patient presents to your clinic with intermittent nausea and vomiting for the past 5 days. She feels well otherwise. What should the nurse practitioner order initially? Electrolytes and qualitative beta hCG A patient has a positive pregnancy test that she performed from an over-the-counter kit. It has been 6 weeks since her last menstrual period. What are the chances that she is pregnant? > 90% The most effective way to decrease the incidence of spina bifida in pregnant patients is to: increase folic acid. A mother has a negative rubella titer. She is not pregnant but is breastfeeding her 4- month-old infant. Is the mother able to safely receive the MMR immunization today? Yes, the immunization offers no risk to her infant. A pregnant patient complains of urinary frequency and dysuria. A urinalysis was obtained. What course of action is most Order urine culture and begin antibiotics appropriate? A patient has type 2 diabetes. Screening for gestational diabetes should: not take place. A pregnant patient has a routine visit that includes a urinalysis. She denies any urinary complaints. Which choice below demonstrates an appropriate action on the part of the nurse practitioner? Order urine culture and begin antibiotics Which factor below confers the lowest risk of ectopic pregnancy? Age less than 18 years A pregnant patient has the following urinalysis report. What is the appropriate course of action? Prescribe nitrofurantoin (Macrobid) A 30-year-old with Type I diabetes has become pregnant. The routine diabetic screening: can be eliminated. Which form of birth control would be the best choice in a lactating mother who wanted to insure that she did not become pregnant? Injectable progestin A pregnant patient asks why she must take will strengthen the calcium during pregnancy. The nurse practitioner replies that it: bones and teeth in your fetus. A pregnant patient has asymptomatic bacteriuria. What is the likely pathogen? E. coli When do the clinical manifestations of an ectopic (fallopian tube) pregnancy typically appear? 6-8 weeks after the LMP HIV screening during pregnancy: is recommended by many learned authorities. When should folic acid be initiated in a female patient contemplating pregnancy if she is not taking it at this time? Now A 17-year-old female is found to be pregnant. What is the most important part of her initial screening? STDs and HIV Which of the following is indicative of changes that occur in the second trimester of pregnancy? BP decreases; HR increases A pregnant patient is likely to have: a venous hum murmur and an S3. Which activity does NOT place the pregnant patient at risk of contracting toxoplasmosis? Eating sushi A patient has a positive pregnancy test that she performed using an over-the-counter kit. What are the chances that a serum pregnancy test will be negative? Almost none The most effective way to decrease neural tube defects is to prescribe folic acid. How much is needed daily for a normal risk pregnancy? 0.4 mg Persistent, intractable vomiting during pregnancy is: hyperemesis gravidarum. What component of a prenatal vitamin is intended to prevent neural tube defects? Folic acid A pregnant patient is 30 weeks’ gestation. She wants to travel to Colorado to go on a hiking trip. She will fly in an airplane. The nurse practitioner knows: travel to a city of high altitude can precipitate preterm labor. Professional Issues Disorders How would you create a therapeutic relationship with a patient? Ask open-ended questions. Certification: validates competence. The research design that provides the strongest evidence for concluding causation is: randomized controlled trials. A liability policy that pays claims only during the period that the policy is active is termed: claims made policy. A nurse practitioner has worked for a large hospital as an RN. As a new nurse practitioner, she has developed a nurse practitioner managed clinic for hospital employees and is employed by the hospital. This nurse practitioner is described as a(n): intrapreneur. A patient you are examining in the clinic states that he has Medicare Part A only. What does this mean? Only hospital visits are covered. Mr. Bowers, a 97-year-old, is not able to make an informed decision due to mental incapacity. He does not have advanced directives, only a durable power of attorney (DPA). How should the nurse practitioner proceed? Allow the DPA to make a decision based on the patient’s known values. A nurse practitioner examined a patient who had been injured by a cat. A 4-centimeter gaping The act of suturing this type of wound represents laceration was present on the patient’s forearm. The nurse practitioner sutured the laceration. The patient subsequently became infected, needed hospital admission, and required IV antibiotics with incision and drainage. How can this situation be characterized? malpractice. The nurse practitioner is examining an older adult with dementia. She is noted to have bruises on her arms and on her posterior thoracic area. The nurse practitioner suspects elder abuse, but cannot be certain. The daughter of this older adult is her caregiver. The daughter is a patient of the nurse practitioner. What should the nurse practitioner do? Report it to the appropriate authorities. The name given to subjects in a research study who do not have the disease or condition being studied, but who are included in the study for comparison are: controls. A nurse practitioner gives a patient 2 weeks of sample medications that will be taken once daily by the patient. The sample medications are packaged by the drug manufacturer. The nurse practitioner’s actions are an example of: dispensing. Prescriptive authority: varies from state to state. A patient you are caring for in your clinic has His medicare benefit covers Medicare Part B. What does this mean? outpatient services. In a research study, the difference between the smallest and largest observation is the: range. The nurse practitioner decides to study a group of patients who are trying to quit smoking. They all will be taking the same type of medication for 42 days to help them stop smoking. The patients have agreed to return to the clinic once weekly for the study’s duration. This type of study design is termed: cohort study. A nurse practitioner (NP) works in an HIV exclusive practice. In talking with a patient, the NP learns that the patient’s sister lives next door to the NP. When the NP sees her neighbor (the patient’s sister), the NP states that she met her sister in the clinic today. The neighbor replies, “Don’t you work in an HIV clinic?” How can this situation be characterized? This is a breach of confidentiality. A nurse practitioner’s scope of practice is influenced by a number of factors. Which one does not influence scope of practice? Court of law A nurse practitioner examined a patient who had been bitten by her husband during an assault. There were numerous bite marks and lacerations on the patient’s forearms. The nurse practitioner sutured the lacerations, but this was contraindicated because This is negligence. of the highly infectious nature of human bites. The patient suffered no ill effects after suturing. How can this be described? A nurse practitioner is working in a minor care area of an emergency department. An illegal immigrant has a puncture wound caused by an unknown sharp object in a trash container. A dirty needle is suspected. The nurse practitioner: should prescribe appropriate medications for HIV exposure even though the nurse practitioner knows the patient can’t afford them. Licensure is: used to establish a designated level of professional competence. A nurse practitioner is working in a minor care clinic. She realizes that a patient with a minor laceration does not have insurance and is using his brother’s insurance information today so that his visit will be covered. How should she proceed? She should let the clinic's business office know what is happening. A nurse practitioner knows that she is HIV positive. She is employed in a private clinic and performs wellness exams on ambulatory adults. The nurse practitioner is: is under no obligation to inform anyone. You are volunteering at a clinic that cares for homeless patients. What’s the most important aspect of a patient’s first visit? Establish trust An older adult male with moderately severe dementia presents with his caregiver daughter. His BMI is 18. His clothes have food stains on them and he looks as though he hasn’t been bathed in days. How should the nurse practitioner handle this? The NP should report this as potential elder abuse. A nurse practitioner is volunteering in a homeless clinic to gain clinical experience. Which statement is true about this? Malpractice insurance is needed by the nurse practitioner. The legal authority to practice as a nurse practitioner in any state is determined by: state legislatures. Which study listed below is considered an experimental study? Meta-analysis What would be the study of choice to determine the cause of a cluster of adult leukemia cases found in an isolated area of a rural state? Case control An older adult was screened for colorectal cancer and had a positive screen. She went on to have a colonoscopy that was normal. She does not have colorectal cancer. The screen was a: false positive. A nurse practitioner has agreed to participate in the Medicare health insurance program. Medicare paid 80% of the charges billed for a clinic visit. What can be done about the other 20% that is owed? The NP can bill the patient for a percentage of the remainder. The Framingham study of cardiovascular disease initiated in the early 1970s is an example of a: cohort study. Anne and Laura work as nurse practitioner (NP) partners in an NP practice. Anne learns that Laura frequently changes patient’s narcotic prescriptions to a different dosage, and then requests that the patient give her the remaining narcotic medications. Some patients have called Anne aside to tell her of this. How can Laura’s actions best be characterized? This appears to be diversion of medications by Laura. Standards of practice are established to: regulate and control nurse practitioner practice. A liability policy that pays claims even after the policy is no longer active is termed: tail coverage. A nurse practitioner is taking care of a patient with health insurance and allergic complaints. The NP is aware that the patient is not using the prescribed allergy medication for her. Instead, the patient is giving the medication to her husband because he does not have insurance. What should the NP do? Only prescribe the medication if the patient promises to use it. A nurse practitioner is working in a minor care area of an emergency department. A patient without insurance arrives with a puncture wound caused by an unknown sharp object in a trash container. A dirty needle is suspected. The nurse practitioner: should prescribe a tetanus shot if needed and appropriate medications for HIV exposure even if he can’t afford them. The Medicaid health program is: funded by both state and federal governments. Anne and Laura work as nurse practitioner (NP) partners in an NP practice. Anne learns that Laura frequently changes patient’s narcotic prescriptions to a different dosage, and then requests that the patient give her the remaining narcotic medications. Laura admits to Anne that she keeps the medications and sometimes takes them herself and occasionally gives them to patients who can’t afford medications. What is Anne’s first professional responsibility? Report Laura’s actions to the state board of nursing. Who certifies nurse practitioners? A nurse practitioner certifying body Psychosocial Disorders Abuse during pregnancy: tends to occur throughout the pregnancy. A patient with bipolar disease has purchased a $10,000 baby grand piano. He does not play the piano. This behavior is typical during: mania. Serotonin syndrome may result from taking an SSRI and: dextromethorphan. A patient reports that she takes kava kava regularly for anxiety with good results. What should the NP evaluate? Liver function studies A 19-year-old college student is at least 15% below her ideal body weight. She reports doing well in classes but drinks alcohol nightly, and several cups of coffee throughout the day. She is bradycardic and gets dizzy when she stands. What may also be observed in this patient? Amenorrhea A patient who abuses alcohol will probably exhibit: elevated ALT, AST, and GGT. An adolescent patient presents with complaints of palpitations and low energy. Her BMI is 16.5 and she is amenorrheic. She has a history of anxiety. The nurse practitioner CBC, glucose, electrolytes, BUN, Cr, urine hCG suspects anorexia nervosa (AN). Which laboratory studies should the NP initially order? A patient is taking a generic version of a selective serotonin reuptake inhibitor (SSRI). She reports intermittent nausea and mild headache daily since she started this medication 5 days ago. How should the nurse practitioner respond? These are typical complaints of patients who take SSRIs. An 80-year-old adult has begun to use over- the-counter diphenhydramine to help him fall asleep. What common side effect can occur in older adults with use of this medication? Next day sleepiness What are the recommendations for screening older patients for depression? Screen at each visit. A 19-year-old college student with anorexia with a BMI of 16 and is being treated as an outpatient. Today she is bradycardic and occasionally has orthostatic hypotension. What might accompany today’s findings? Amenorrhea Which older adult is at highest risk of suicide? 86-year-old male with chronic pain and depression Four days ago, a 79-year-old female lost her husband of 55 years. She presents today with her daughter because she believes that she is “going crazy”. She reports that she often hears her husband's voice though she realizes that he has died. She has not slept well since his death and hasn’t eaten much. She has taken her usual medications for hypertension, osteoporosis, osteoarthritis, and hypothyroidism. She has no history of psychiatric illness. How should the nurse practitioner manage this? Tell her that this is a normal response and that it will resolve. A patient has been diagnosed with anxiety. What sleep disturbance might she have? Difficulty falling asleep A 62-year-old patient started taking paroxetine 1 week ago for depression. She calls to report intermittent headache and nausea. What is a likely etiology? Drug side effect A common side effect of trazodone may be alleviated by: taking this medication at bedtime. Which patient is most likely to exhibit depression related to his illness? A patient with: Parkinson’s disease. The nurse practitioner is treating a 22-year-old Let the obstetrician and for depression with high-dose sertraline. After several months of dosage changes, she is finally doing well and comes today for a follow- up visit. She is happy and states that she might be pregnant. A urine test indicates pregnancy. The NP has referred the patient to an obstetrician who will see the patient in 4 weeks. How should the sertraline be handled today? patient make a decision about continuing sertraline. A 76-year-old depressed patient is started on an SSRI. When should another antidepressant be tried if there is no response? 8-12 weeks What is the usual age of onset of symptoms for patients with bipolar disorder? Between 15 and 30 years Delirium differs from dementia because delirium: often develops acutely. The clinical difference between minor depression and major depression is: the number of symptoms present. Carbamazepine is used in patients with bipolar disorder for mood stabilization. Prescribers who have patients taking carbamazepine should be alert to: drug-drug interactions. A patient with bulimia nervosa probably has concurrent: anxiety. A patient with acute anxiety will experience the fastest relief of symptoms when he is treated with: a benzodiazepine. A patient may derive benefit from a tricyclic antidepressant (TCA) if he experiences depression and: chronic pain. Which statement about bulimia nervosa is accurate? High dose SSRIs are used to treat this. A depressed patient is started on an SSRI. When should another antidepressant be tried if there is no response? 8-12 weeks MMSE helps to identify patients with symptoms of: dementia. How should the nurse practitioner approach a patient who consumes excessive amounts of alcohol but denies that he has a problem? Tell him that you are concerned about his health. Which of the following characterizes bulimia nervosa? Binge eating In patients who exhibit depression, selective serotonin reuptake inhibitors (SSRIs) are commonly chosen as a medication for treatment. SSRIs are often chosen because they: are safer than other medications for depression. Depression is diagnosed on clinical presentation. What time frame is important for distinguishing between depressed mood and clinical depression? 2 weeks Which patient is at highest risk of suicide? 86-year-old male with chronic pain Mrs. Smith has recently lost her job. She is unable to sleep at night because “she is just so worried she can’t sleep.” Which medication listed below should NOT be used to treat anxiety? Zolpidem (Ambien) An 86-year-old male reports feelings of anhedonia for the last month. What should be part of the nurse practitioner's initial assessment? Suicidal ideations An adolescent female patient with anorexia nervosa must exhibit 4 criteria for diagnosis. Which criterion listed below is NOT part of the diagnostic criteria? Absence of menstrual cycle A patient with an eating disorder may concomitantly exhibit: anxiety disorders. The preferred medication class to treat patients with an initial episode of minor depressive disorder is: selective serotonin reuptake inhibitors. A newly diagnosed pregnant teenager has suspected depression. Before a diagnosis is made, she should have a CBC, TSH, renal and liver function tests and: urine toxicology screen. A good first choice of antidepressants in an older adult is: an SSRI. Serotonin is thought to play a role in the etiology of: depression. Older adults who are treated for depression with tricyclic antidepressants (TCAs) often exhibit: arrhythmias. Which screen for alcohol abuse has been validated in older adults? CAGE A 34-year-old bipolar patient has been placed on a fluoxetine and valproate for manic depressive symptoms. He has had great Valproate levels, platelet count, liver function studies improvement in his symptoms and has returned to work. The psychiatrist has released him to your care. What must be monitored in this patient? A nurse practitioner suspects that a patient is abusing ethanol. What laboratory values would support this suspicion? Elevated ALT, AST, and GGT A patient presents to your clinic numerous times with vague complaints. She seems to respond poorly to medical treatment that is given to her. What should be considered when obtaining a history from her? Physical abuse or depression Which depressed patient below has characteristics that are risk factors for suicide? 78-year-old male recently widowed Which criterion below is a criterion for Alzheimer’s disease? Impairment of executive function Tricyclic antidepressants may be safely used in older patients who have: hypothyroidism. A 6-year-old is brought to your clinic because of behavior problems at school. DSM V criteria are used to diagnose attention deficit disorder (ADD). Which finding is likely present? Inattention Which findings suggest that a patient may be abusing alcohol? Macrocytosis, tremulousness, hypertension A male patient has a family history of bipolar disorder in two first-degree relatives. Bipolar disorder: often affects multiple family members. The major advantage of the CAGE questionnaire is: brevity of questions. A common strategy used to minimize the incidence of side effects when giving an older adult a selective serotonin reuptake inhibitor (SSRI) is: prescribe a low dose initially. A patient presents to the nurse practitioner’s clinic and states that she feels sad and thinks she’s depressed. What information is important to elicit in order to diagnose her with depression? How long have you felt like this? A 29-year-old postpartum female reports that she is having difficulty with concentration, sleep, and has feelings of guilt. She states that she feels sad most of the time. These symptoms have been present since the birth of her baby about 1 month ago. She can be diagnosed with: postpartum depression. A 70-year-old male patient has an elevated MCV with an anemia. His triglycerides are 420. What should be suspected? Alcohol abuse A patient with an eating disorder might exhibit evidence of: anxiety disorders. Medications considered first line to treat attention deficit disorder and attention deficit hyperactivity disorder are Schedule: II. A 4-year-old is being examined today in the NP clinic. He appears shy and does not make eye contact with the examiner. The mother does not make eye contact with the examiner either. The patient lacks animation and does not smile. What likely possibility must be considered? Neglect is possible. A patient has suspected serotonin syndrome. How can this be diagnosed? Clinical exam and index of suspicion Which statement describes depression in older adults? They can be managed with some of the same medications as younger adults. The most common mental disorder in older adults is: anxiety. Which statement about attention deficit disorder (ADD) is correct? DSM V is used to diagnose a child with ADD. The most common co-morbidity associated with depression is: anxiety. A 38-year-old patient diagnosed with bipolar disease has taken lithium for many months. His mood has stabilized. He was told to report frequent urination while taking lithium. What might be the underlying cause of his frequent urination? Diabetes insipidus Within 6 months of treatment, patients who are treated for depression with selective serotonin reuptake inhibitors often exhibit: weight gain. Which symptom listed below is typical of depression? Early morning wakening Respiratory Disorders produces bronchodilation. The major laboratory abnormality noted in patients who have pneumococcal pneumonia is: leukocytosis. An 80-year-old has Stage 3 COPD. He is most likely to have concomitant: anxiety or depression. A 20-year-old college age student has a positive TB skin test. Which choice listed below provides definitive diagnosis of tuberculosis? A sputum specimen A 60-year-old patient newly diagnosed with COPD presents to your office. He would like to get the influenza immunization. He has no evidence of having had the pneumococcal immunization. What statement is correct? He should receive both influenza and pneumococcal immunizations today. A patient received the pneumonia immunization at age 60 years. He is 65 years old and presents to your clinic today. What recommendation should be made about the pneumococcal immunization? He should receive another one today. Mild persistent asthma is characterized by: symptoms occurring more than twice weekly. An uncommon symptom associated with acute bronchitis is: temperature > 101°F. A 44-year-old nonsmoker is diagnosed with pneumonia. He is otherwise healthy and does not need hospitalization at this time. Which antibiotic can be used for empirical treatment of pneumonia according to the most recent Infectious Diseases Society of America/American Thoracic Society guidelines? Azithromycin What disease is usually managed with a short-acting or long- acting inhaled anticholinergic medication? COPD How should a 20-year-old college age student who presents with cough, night sweats, and weight loss be screened for TB? A TB skin test A patient has cough, pharyngitis, nasal discharge, and fever. He has been diagnosed with acute bronchitis. Which symptom is least likely in the first 3 days of this illness? Cough A 65-year-old patient has COPD. She receives a prescription for an albuterol inhaler. What medication information should be provided to this patient? This may cause tachycardia A patient with COPD has been using albuterol with good relief for shortness of breath. He is using it 3-4 times daily over the past 4 weeks. How should the NP manage this? Add a long-acting beta agonist A 26-year-old being treated for community-acquired pneumonia (CAP) has been taking azithromycin (standard dose) in therapeutic doses for 72 hours. His temperature has gone from 102° F to 101° F. What should be done? Stop azithromycin and initiate a respiratory quinolone A 75-year-old female with emphysema who has been treated with inhaled steroids for many years should: should be screened for osteoporosis. The most common symptom associated with acute bronchitis is: cough. The most common sequela of influenza in older adults is: pneumonia. A patient recently received levofloxacin for 7 days to treat pneumonia. His respiratory symptoms have resolved, but today he calls the office. He reports having severe watery diarrhea, abdominal cramping, and low-grade fever. What should be done? Order a stool specimen The most common cause of atypical pneumonia in adults is: Mycoplasma pneumoniae. An 83-year-old healthy adult is diagnosed with pneumonia. He is febrile but in no distress. What is the preferred treatment for him? Levofloxacin Breath sounds auscultated over the periphery of the lung fields are quiet and wispy during the inspiratory phase followed by a short, almost silent expiratory phase. These breath sounds are considered: vesicular. A patient with acute bronchitis was diagnosed at an urgent care center 10 days ago. He reports that he received an antitussive for nighttime cough, a steroid injection and oral steroids, and an antibiotic. Which of these interventions was of greatest benefit in None of them resolution of his symptoms? A 60-year-old patient reports chronic cough and sputum production. He has a long history of exposure to secondhand cigarette smoke from his wife. What diagnosis is most likely? COPD Mr. Smith has smoked for 45 years. Which of the following medications may worsen one of his diseases? Propanolol What does a peak flow meter measure? Expiratory flow A 78-year-old adult who has a 50 pack year smoking habit asks the nurse practitioner about the benefits of quitting “at my age.” What should the nurse practitioner reply? This will decrease your risk of all cause mortality 5 years after stopping. A patient with asthma uses one puff twice daily of fluticasone and has an albuterol inhaler for PRN use. He requests a refill on his albuterol inhaler. His last prescription was filled 5 weeks ago. What action by the NP is appropriate? Increase the dose of the inhaled steroid, refill the albuterol A patient who has asthma presents with chest tightness, wheezing, coughing, and fever. He has wheezing and diminished breath sounds in the upper right lobe. His cough is nonproductive, and he denies nasal symptoms. Which symptom is not likely related to his asthma? Fever A patient who has asthma presents with chest tightness, wheezing, coughing, and fever. He has wheezing and diminished Fever breath sounds in the upper right lobe. His cough is non- productive, and he denies nasal symptoms. Which symptom is most likely related to pneumonia? Which medication below should be avoided in a patient with stage 3 COPD? Cough suppressant with codeine Which drug class is never used to treat chronic obstructive pulmonary disease (COPD)? Leukotriene blockers Patients with asthma: can cough or wheeze. A 65-year-old has been diagnosed with asthma. Older patients who have newly diagnosed asthma: are more likely to cough. Patients who have cough-variant asthma: all exhibit cough. M. pneumoniae and C. pneumoniae are respiratory pathogens that: cause atypical pneumonia. “Good control” of asthma is measured by the number of times weekly a patient uses a rescue inhaler. What choice below indicates “good control”? Once weekly A patient with acute bronchitis and cough for 5 days calls to report that his cough is productive of discolored sputum. He has no other new symptoms. How should the nurse practitioner Continue the original plan of care. manage this? Mycoplasma pneumoniae is: a disease with extrapulmonary manifestations. An 18-month-old child is diagnosed with bronchiolitis. His respiratory rate is 28 breaths per minute. Which choice below is most appropriate for patient management? Antipyretics An adult has upper respiratory symptoms and cough for the past 14 days. What should be considered? Pertussis A 24-year-old college student who does not smoke is diagnosed with pneumonia. He is otherwise healthy and does not need hospitalization at this time. What antibiotic represents the best choice for treatment for him? Clarithromycin Which of the following is NOT part of the differential for a patient who complains of cough? Obesity Which of the following characteristics is always present in a patient with COPD? Obstructed airways A 6-year-old child who has moderate persistent asthma is diagnosed with pneumonia after chest X-ray and laboratory studies. He developed a sudden onset of fever with chills. He is in no distress. What is the preferred treatment for him? Amoxicillin A 24-year-old presents with fever, rhinorrhea, and paroxysmal, high-pitched cough. This is: pertussis. A patient has received a prescription for lisinopril. Which side effect most commonly occurs with this medication? Dry cough The pneumococcal immunization in infants has: shifted the pathogenesis to fewer cases of S. pneumoniae. A patient presents with symptoms of influenza during influenza season. He has not received the immunization against influenza. What should be used to help diagnose influenza in him? A nasal swab The most common cause of pneumonia in people of all ages is: S. pneumoniae. Which of the following medications should be used cautiously in a patient who has asthma? Timolol ophthalmic drops Which medication below is contraindicated in an asthma patient because it may increase risk of sudden death if used alone? Long-acting bronchodilator A patient with cough and fever is found to have infiltrates on chest x-ray. What is his likely diagnosis? Pneumonia An example of a short-acting beta agonist is: levalbuterol. A healthy 7-year-old child is diagnosed with atypical pneumonia. He is febrile but not in distress. What is the preferred treatment for him? Azithromycin Which patient might be expected to have the worst FEV1? A 65-year-old with emphysema The most common cause of pneumonia in an otherwise healthy 3-year-old child is: a viral infection. A patient with pneumonia reports that he has rust-colored sputum. What pathogen should the nurse practitioner suspect? Streptococcus pneumoniae The gold standard for diagnosing pneumonia on chest X-ray is the presence of: infiltrates. Which of the following may be used to diagnose COPD? Pulmonary function tests or spirometry A patient is diagnosed with asthma. Which question is most important to ask when deciding on medication management? How often do your symptoms occur? A 67-year-old patient with COPD presents an immunization record that reflects having last received the pneumococcal immunization (PPSV23) when he was 60 years old. Which statement below reflects the current standard of practice recommended by CDC for this patient? He should be vaccinated today with PCV13. Which choice below most appropriately differentiates acute bronchitis from pneumonia in a patient who has a productive cough? Chest X-ray A 30-year-old patient with intermittent asthma is using his “rescue” medication once daily. How should this be managed? He should receive a prescription for a(n): an inhaled steroid. The chest circumference of a 12 month-old is: equal to head circumference. STD Disorders A patient was exposed to HIV through sexual intercourse last night. He was screened and found to be negative for HIV. When should he be screened again? 4-6 weeks A female patient has been diagnosed with chlamydia. How should this be managed? Treat with azithromycin An example of primary prevention is: using a condom to prevent infection with an STD. The risk of HIV transmission is increased: when other STDs are present. A female patient and her male partner are diagnosed with trichomonas. She has complaints of vulval itching and discharge. He is asymptomatic. How should they be treated? They both should receive metronidazole. A 30-year-old male who is sexually active complains of pain during bowel movements. The digital rectal examine is negative for hemorrhoids, but the prostate gland is tender. What should be suspected? Acute bacterial prostatitis A 24-year-old female presents with abdominal pain. What additional finding supports a diagnosis of pelvic inflammatory disease (PID)? Cervical motion tenderness In a private NP clinic, a patient presents with trichomonas. State law requires reporting of STDs to the public health department. The patient asks the NP not to report it because her husband works in the public health department. How should this be managed by the NP? Report it to public health as required by law Which of the following statements regarding HIV is correct? There are few conditions that cause depletion of CD4 cells other than HIV A 25-year-old female presents with lower Temperature > 101°F abdominal pain. Which finding would indicate the etiology as pelvic inflammatory disease? A 27-year-old asymptomatic male presents with generalized lymphadenopathy. He has multiple sexual partners and infrequently uses condoms. Of the following choices, what test should be performed? HIV test Which of the following describes the most common clinical presentation of trichomonas in a male? No clinical symptoms A male with gonorrhea might complain of: dysuria. Which risk factor has the greatest impact on HIV transmission? Viral load A 35-year-old patient is HIV positive. Which finding on an oral swab may be indicative of thrush? Yeast A 26-year-old female patient has been diagnosed with gonorrhea. How should she be managed? Ceftriaxone and azithromycin A patient with newly diagnosed genital herpes would appropriately receive a prescription for: famciclovir. A patient requests screening for HIV after a sexual exposure. What are CDC’s recommendations for screening for this patient? She should be screened today, with repeat screening at 4- 6 weeks, and 3 months The greatest risk of transmitting HIV is during: high viral load periods. A patient being treated for trichomoniasis receives a prescription for metronidazole. What instructions should she be given? Alcohol should be avoided when taking this medication Syphilis may present as: a rash. How should a patient with suspected syphilis be screened? A serum assessment A 21-year-old female presents with six 0.5-cm human papillomavirus (HPV) lesions on her vulva. An appropriate treatment option for this patient would be: podophyllin. How long should a patient be treated with antibiotics if he has prostatitis secondary to an STD? 14 days or longer A 32-year-old female was exposed to HIV after sexual intercourse. When do the majority of Within 3 months patients seroconvert if they are going to do so? A male patient presents with dysuria and penile discharge. He states that his female partner has an STD, but he is not sure which one. Which of these should be part of the differential? Chlamydia and gonorrhea A 26-year-old HIV-positive patient presents with photophobia and temperature of 103.2° F. He complains of a headache. On exam, he is unable to demonstrate full extension of the knee when his hip is flexed. Which choice below is the most likely diagnosis? Meningitis Chancroid is considered a cofactor for transmission of: HIV. A 23-year-old female who is homeless presents to the free clinic. She should be screened for: TB, HIV, and hepatitis. A sexually active adolescent male reports that he has dysuria. How should this be managed? Urethral/urine cultures should be collected and screening done for STDs Urology Disorders A 73-year-old male patient reports that he is experiencing a weakened urinary stream, urinary frequency, and urgency. He is waking up once or twice nightly to urinate. How should the nurse practitioner proceed?DRE, urinalysis, PSA DRE, urinalysis, PSA A 44-year-old female patient is diagnosed with a urinary tract infection (UTI). Which bacteria count collected via midstream, clean catch supports a diagnosis of UTI? > 100,000 bacteria The most common pathogen found in patients with pyelonephritis is: E. coli. Ciprofloxacin given to treat a urinary tract infection would be contraindicated in a: pregnant patient. An adolescent has suspected varicocele. He has dull scrotal pain that is relieved by: recumbency. A 76-year-old male presents with urethral irritation after voiding. If sexually transmitted diseases and urinary tract infection are ruled out, what is another etiology? Chronic prostatitis A healthy 32-year-old female has left flank pain and nausea. What is the most likely diagnosis? Pyelonephritis A patient with urolithiasis is more likely to: be of male gender. A patient comes to clinic today with a complaint of green urine for the past 6 hours. She feels well otherwise. The most likely reason for this is: something she has consumed. An 86-year-old male has symptomatic benign prostatic hyperplasia (BPH). What choice may exacerbate the symptoms of BPH? Furosemide Testicular torsion can produce: scrotal edema. A sexually active male patient presents with epididymitis. What finding is likely? Recent history of heavy physical exercise Mrs. Jackson complains of urinary incontinence when she laughs or sneezes. What should be used first line to treat her symptoms? Kegel exercises A 25-year-old male patient with subacute bacterial epididymitis should be treated initially with: doxycycline. A female patient with a complaint of dysuria has the following urinalysis results. The most appropriate diagnosis is: UTI with hematuria. A 79-year-old female patient with urinary frequency is found to have a UTI. What medication could produce arrhythmias in her? Ciprofloxacin Which of the following choices may result in an INSIGNIFICANT increase in prostate specific antigen (PSA)? Digital rectal exam A patient has a urinary tract infection. What findings on urine dipstick best describe a typical urinary tract infection? Positive leukocytes, positive nitrites A patient has urinary burning, frequency, and urgency. A likely explanation is that the patient: has a sexually transmitted disease. A 31-year-old female patient present with fatigue, fever (101° F), and worsening unilateral low back pain for the past 5 days. Her pain is 5/10 on the pain scale and has been unresponsive to ibuprofen. She denies abdominal pain, but is anorexic and nauseous. She denies vaginal discharge. What should be included in the differential diagnosis? Pyelonephritis A patient has been diagnosed with pyelonephritis. He probably will have: costovertebral angle tenderness. An example of a drug that targets the renin-angiotensin- aldosterone system is a(n): ACE inhibitor. A male patient with lower urinary tract symptoms has the following urinalysis. What medication should be given and for how many days? Trimethoprim- sulfamethoxazole (TMPS) for 7-10 days The most common pathogen associated with pyelonephritis is: E. coli. The incidence of pyelonephritis is: less common than urinary tract infections. A 70-year-old male patient is diagnosed with renal disease. Which activity will help preserve kidney function? Daily increase in water consumption A patient with dysuria has a urine specimen that reveals < 10,000 bacteria and numerous trichomonads. How should this patient be managed? Metronidazole for 7 days A 24-year-old female patient has been diagnosed with an uncomplicated UTI. Which assessment below is least important at this time? Vaginal exam The most appropriate time to begin screening for renal nephropathy in a patient with Type 2 diabetes is: at diagnosis. A 50-year-old male patient reports that he has a sensation of scrotal heaviness. He reports that the sensation is worse at the end of the day. He denies pain. What is the likely etiology of these symptoms? Inguinal hernia Which medication should be avoided in a patient with a sulfa allergy? Sulfamethoxazole A female patient who is 45 years old states that she is having urinary frequency. She describes episodes of “having to go right now” and not being able to wait. Her urinalysis results are provided. What is part of the differential? Diabetes A 6-month-old male has a palpable cystic mass in his scrotum. His Referral to urology if mother states that sometimes the size of his scrotum varies. What should the nurse practitioner do next? this has not resolved in 6 more months A 22-year-old male patient has symptoms of burning with urination. Which assessment below is least important at this time? Abdominal exam A male child has swelling in the upper left thigh below the inguinal ligament. He complains of cramping and intermittent lower abdominal pain. What should be included in the differential diagnosis? Femoral hernia Which patient is most likely to develop a urinary tract infection? 20-year-old sexually active female A 26-year-old female presents with flank pain that waxes and wanes. What finding on urinalysis should direct the nurse practitioner's next action? Blood How long should a female patient with an uncomplicated UTI be treated with an oral antibiotic? 3 days The term that describes the urethral opening on the ventral surface of the penis is: hypospadias. A pregnant patient with urinary frequency is found to have a UTI. Which drug is safest to treat this? Nitrofurantoin A 72-year-old male patient has early renal insufficiency. What laboratory test best assesses his kidney function? Glomerular filtration rate A physically independent 75-year-old was diagnosed with mild cognitive impairment 6 months ago. She resides in an assisted living facility. She is in the clinic today for a scheduled visit. Her adult daughter reports that about 2 weeks ago her mother had an episode of urinary incontinence, but no known episodes since. She is found to have asymptomatic bacteriuria. How should she be managed? Monitor her for symptoms of a urinary tract infection. Which of the following increases the risk of cryptorchidism? Premature birth What early finding may lead the provider to suspect renal artery stenosis in a 3-year-old male? Increased blood pressure An adolescent male has had a sudden onset of severe scrotal pain following a kick in the groin earlier in the day during a soccer game. How should this be managed? He should be referred to the emergency department. A sexually active adolescent male has a warty growth on the shaft of his penis. It is painless. This is likely: human papilloma virus. A patient reports that he has been taking saw palmetto. He is using this because he thinks it: improves urine flow. Women’s Health Disorders “Hot flashes” that occur during menopause are thought to be related to: fluctuating progesterone levels. Women who use diaphragms for contraception have an increased incidence of: urinary tract infection. Which type of incontinence would NOT benefit from pelvic floor muscle training? Overflow When collecting cervical cells for a PAP smear, when are the endocervical cells typically collected? After the ectocervical specimen with a brush What choice below has no precautions for oral contraceptive pill use? Varicose veins Three of the following interventions are appropriately used to prevent osteoporosis after menopause. Which one is not? Estrogen replacement therapy The recommended time to initiate screening for cervical cancer in women is: at age 21 years. The primary risk factor for development of breast cancer in women of average risk is: age. The frequency for cervical screening depends on the patient and her age. What is the longest recommended time interval between cervical screens for patients who are 5 years 30-64 years old? The frequency for cervical screening depends on the patient and her age. What is the longest recommended time interval between cervical screens for patients who are 21-65 years of age? 5 years The first step in evaluating a breast lump is: history and physical exam. The clinical syndrome resulting from replacement of normal vaginal flora with anaerobic bacteria is: bacterial vaginosis. Clue cells are found in patients who have: bacterial vaginosis. Athletic amenorrhea increases the risk of: osteoporosis. An initial pharmacologic approach to a patient who is diagnosed with primary dysmenorrhea could be: NSAIDs at the time symptoms begin or onset of menses. An 84-year-old female patient is a resident in an assisted living facility. She has early dementia. She walks daily and has had urinary incontinence for years. She has loss of urine with coughing, sneezing, and if she is unable to get to the bathroom quickly enough. Her urinary incontinence is likely to be: mixed. After a vaginal exam, a patient received a prescription for metronidazole 500 mg twice daily for 7 days. What was her likely diagnosis? Bacterial vaginosis According to the American Cancer Society (ACS), what recommendation should be made to a 70-year-old female regarding mammograms if she is considered low risk? She should have them every 1-2 years as long as she has a reasonable life expectancy. A young female has breast buds bilaterally. This represents Tanner Stage: II A young female adult presents with vaginal discharge and itching. Besides trichomoniasis and yeast, what else should be included in the differential? Bacterial vaginosis A patient who takes oral contraceptive pills is at increased risk of: gallbladder disease. A patient who is scheduled for pelvic exam with PAP smear should be advised to avoid douching, sexual intercourse, and tampon use before her exam. For how long should she be advised to avoid these activities for optimal evaluation? 48 hours A patient who is 35 years old has identified a small, discrete mass in one breast. How should this be evaluated initially? Ask whether the mass changes at the time of menses. A patient asks the NP’s advice about an herb to help with her hot flashes. The NP knows these: may be contraindicated in patients with history of breast cancer. A nurse practitioner identifies the following image during a microscopic exam of vaginal discharge. These are probably: hyphae. A female should be told to take her oral contraceptive pill (ocp) at bedtime if she experiences: nausea. A female patient who takes oral contraceptives has just completed her morning exercise routine. She complains of pain in her right calf. Her blood pressure and heart rate are normal. She is not short of breath. Her calf is red and warm to touch. What is NOT part of the differential diagnosis? Sciatica A female patient is 35 years old. She has never had an abnormal PAP smear and has had regular periodic screening. If she has a normal PAP smear with HPV testing today, when should she have the next cervical cancer screening? 5 years A female patient complains of dysuria with vaginal discharge. How should she be managed? Perform an abdominal exam, urinalysis, and pelvic exam A definitive diagnosis of osteoporosis can be made when: bone mineral density is 2.5 standard deviations below the mean. A 70-year-old female has been in a mutually monogamous relationship for the past 33 years. She has never had an abnormal Pap smear. What recommendation should be made regarding Pap smears for her? They can be discontinued now. A 54-year-old female presents with a small to moderate amount of vaginal bleeding of recent onset. She has been postmenopausal for approximately 2 years. What diagnosis is least likely? Ovarian cancer A 52 year old female presents to your clinic with a palpable mass in her right breast. Her last normal mammogram was 6 months ago. What is true about this lump? It is probably a benign lesion. A 51-year-old female patient presents with a 2-cm palpable breast mass. How should this be evaluated to determine whether it is solid or cystic in nature? Ultrasound A 50-year-old female believes that she is menopausal. She complains of hot flashes and has not had menses in 12 months. Which of the following test results would be expected during menopause? Increased follicle- stimulating hormone A 4-year-old female is brought in to the clinic by her mother, who reports that she is constantly scratching “her private part.” The patient states that it itches. On exam, the vagina is red and irritated. How should the NP proceed? Collect a vaginal swab of the external vagina for microscopic evaluation A 32 year-old female presents to your clinic for her annual GYN exam. A 1cm soft, non-tender, round mass is found at the 4 o’clock position at the posterior vaginal introitus. What is the most likely diagnosis? Bartholin duct cyst A 28-year-old female presents with a slightly tender 1.5-cm lump in her right breast. She noticed it 2 days ago. She has no associated lymphadenopathy and there is no nipple discharge. How should she be managed? Re-examination after her next menses A 24-year-old female patient who is sexually active complains of vaginal itching. If she has bacterial vaginosis, she might complain of: a “fishy” vaginal odor after coitus. A 22-year-old female states that she has multiple sexual partners and inconsistently uses barrier protection. Which form of birth control should the nurse practitioner avoid prescribing in this patient? Intrauterine device A 22-year-old female has been diagnosed with polycystic ovarian syndrome (PCOS). What is a common finding? Elevated insulin levels A 20-year-old female reports that her grandmother and mother have osteoporosis. What should she be encouraged Smoking cessation, to do to reduce her risk of osteoporosis? weight bearing exercise A 17-year-old presents with complaints of dysmenorrhea. Which finding below suggests that this is secondary dysmenorrhea? Dysmenorrhea is not limited to menses A 16-year-old female is diagnosed with primary dysmenorrhea. She has taken over-the-counter ibuprofen in 800-mg increments every 8 hours during menses for the past 3 months, with minimal relief of symptoms. What intervention will provide greatest relief of dysmenorrhea symptoms? Combined oral contraceptives A 14-year-old female has never menstruated. She and her mother are concerned. What is most important for the NP to assess? Tanner stage [Show More]

Last updated: 1 year ago

Preview 1 out of 196 pages

Add to cart

Instant download

document-preview

Buy this document to get the full access instantly

Instant Download Access after purchase

Add to cart

Instant download

Reviews( 0 )

$18.50

Add to cart

Instant download

Can't find what you want? Try our AI powered Search

OR

REQUEST DOCUMENT
306
2

Document information


Connected school, study & course


About the document


Uploaded On

Sep 29, 2021

Number of pages

196

Written in

Seller


seller-icon
A+ Solutions

Member since 3 years

164 Documents Sold


Additional information

This document has been written for:

Uploaded

Sep 29, 2021

Downloads

 2

Views

 306

Document Keyword Tags

Recommended For You

Get more on TEST BANK »

$18.50
What is Browsegrades

In Browsegrades, a student can earn by offering help to other student. Students can help other students with materials by upploading their notes and earn money.

We are here to help

We're available through e-mail, Twitter, Facebook, and live chat.
 FAQ
 Questions? Leave a message!

Follow us on
 Twitter

Copyright © Browsegrades · High quality services·